Vous êtes sur la page 1sur 40

Questions de Quiz connecté

Chapitre 2

Alberto était propriétaire de Blackacre, qui était adjacent à Whiteacre, propriété de


Berta. En 1998, Alberto a construit son garage sans faire de relevé et, par conséquent,
une partie du nouveau garage a dépassé d'environ deux pieds la limite de propriété sur
Whiteacre. En 2005, Alberto est décédé. Dans son testament, il a légué Blackacre à sa
fille Cara. Elle s'y est installée et, en 2007, a décidé de vendre Blackacre à Darrell, qui
s'y est rapidement installé. En 2012, Berta a essayé de vendre Whiteacre à un acheteur
potentiel, qui a découvert que le garage dépassait la limite. En 2012, Berta a intenté un
procès à Darrell pour l'obliger à déménager le garage ou à lui verser des dommages-
intérêts. Darrell revendique un titre de propriété par possession adverse, ce qui
nécessite dix ans de possession dans cette juridiction. Lequel des éléments suivants
est vrai ?

(A) Darrell ne peut pas respecter le délai légal de possession, parce qu'il peut ajouter la
possession de Cara mais pas celle d'Alberto.

(B) Darrell ne peut pas ajouter la possession de Cara ou d'Alberto pour satisfaire à
l'exigence légale.

(C) Darrell devrait être en mesure de satisfaire à l'exigence de l'adverse


possession en utilisant la possession de Cara et d'Alberto en plus de la sienne.

(D) Darrell ne peut en aucun cas se prévaloir d'un droit de possession, car il ne remplit
la condition d'adversité dans aucune juridiction.
Raison d'être :
La bonne réponse estC. Darrell n'a été en possession que pendant cinq ans, de sorte que pour
respecter la période légale de dix ans, Darrell devra ajouter la possession d'Alberto et de Cara. Un
possesseur adverse peut "s'approprier" la possession de possesseurs antérieurs s'ils ont un lien de
connexité, ce qui signifie simplement qu'il existe un lien raisonnable entre eux qui fait qu'il est juste
d'accorder au possesseur actuel le crédit de la possession antérieure. En général, une personne qui
hérite ou se voit attribuer un bien par testament, comme Cara, a ce lien, de même qu'un acheteur de
bonne foi du bien, comme Darrell. Les réponsesA etB sont donc incorrectes. Le rattachement ne
serait autorisé que lorsqu'il n'y a pas de lien entre un possesseur et un autre. Il ne semble donc pas
y avoir de raison pour qu'un tribunal refuse à Darrell le droit d'ajouter ces périodes de possession
antérieures.D est incorrect car Darrell n'a pas l'autorisation d'empiéter sur la propriété de Berta ; il
est donc opposant. Dans certaines juridictions, le fait que Darrell ne savait pas qu'il empiétait sur le
domaine public peut détruire son opposition, mais la plupart des juridictions considèrent que la
possession sans autorisation est une opposition.
Chapitre 3

1.
O a concédé Blackacre "à l'Université tant que le tabac n'est pas consommé sous quelque forme
que ce soit sur les lieux, et si jamais du tabac est consommé, Blackacre reviendra à O, à ses
héritiers, à ses successeurs et à ses ayants droit". Lequel des éléments suivants est vrai ?
(A) L'Université possède un fief simple soumis à une condition ultérieure.

(B) L'université possède un droit simple déterminable.

(C) L'université dispose d'un reliquat conditionnel.

(D) Le droit de l'Université est nul en vertu de la règle contre les perpétuités.

Raison d'être :
La bonne réponse estB. L'université a un intérêt présent, la réponseC est donc incorrecte car le
reliquat est un intérêt futur. Parce que l'intérêt de l'Université peut potentiellement durer
éternellement, il s'agit d'un fief simple. Toutefois, comme il peut prendre fin avant terme, il s'agit
d'une forme de redevance défaisable. Il existe deux possibilités : le fief simple soumis à une
condition ultérieure et le fief simple déterminable. Une succession déterminable a une durée (par
exemple, "tant que" ou "jusqu'à") intégrée dans la concession, tandis qu'une succession
conditionnelle a une condition attachée après une concession absolue ("à condition que" ou "à la
condition que"). En outre, une redevance déterminable est généralement suivie d'une réversion
automatique en cas de violation, tandis que la redevance conditionnelle est assortie d'une
résiliation facultative ("O peut reprendre possession"). Dans ce cas, l'aide est accordée pour une
durée déterminée et la résiliation est automatique. Par conséquent, l'Université possède une
redevance simple déterminable, et la réponseA est incorrecte. La réponseD est incorrecte car le
droit de l'Université n'est pas annulé en vertu de la règle car celle-ci ne s'applique qu'aux droits
futurs (et d'ailleurs, le droit de O n'est pas non plus affecté car les droits conservés par le
concédant ne sont pas soumis à la règle).

2.

Le 16 décembre 2018, le comté de Cook a cédé un terrain "à la ville de Chicago pour la durée de
vie de George Lucas, le reste revenant au Lucas Museum of Narrative Art, mais si ledit terrain
n'est pas utilisé pour une exposition permanente de l'Étoile de la mort originale, il reviendra à son
propriétaire".
Quel intérêt le musée a-t-il acquis ?

(A) Fee simple déterminable


(B) Fee simple sous réserve d'une condition subséquente

(C) Fief simple absolu

(D) Aucune de ces réponses

Raison d'être :
La bonne réponse est(D). Le musée a acquis un droit de passage soumis à un dessaisissement, un
droit futur. Les points(A),(B) et(C) sont incorrects car ils désignent tous trois un droit de
possession présent. Le seul intérêt possessoire actuel dans cette concession est le domaine
viagerpur autre vie à la ville de Chicago pour la durée de vie de Lucas.

Chapitre 5

Après leur mariage en 2010, Juan Carlos et Jamie ont acheté une maison, dont ils ont
pris possession en tant que locataires. Jamie pense maintenant que la maison ne
correspond plus aux besoins de la famille et elle veut déménager. Juan Carlos aime la
maison et n'a aucune envie de déménager. Jamie a demandé conseil à un ami, qui lui a
suggéré d'intenter une action en partage afin de forcer la vente de la maison.
Jamie réussira-t-il ?

(A) Oui, car tout détenteur d'intérêts peut obtenir gain de cause dans le cadre d'une
demande de partage.

(B) Oui, car Juan Carlos refuse de vendre.

(C) Non, car un contrat de location en pleine propriété ne peut être dissocié.

(D) Non, car un contrat de location par l'intégralité peut être dissocié.
Raison d'être :
La bonne réponse est(C) : un contrat de location en pleine propriété, réservé aux couples mariés, ne
peut être rompu unilatéralement par le biais d'une action en partage. Par conséquent, les points(A)
et(D) sont incorrects. Lepoint (B) est également incorrect, car le refus de Juan Carlos de vendre n'a
pas d'incidence sur la restriction relative à la séparation unilatérale des biens détenus par
l'ensemble.
Lecture recommandée :
Pages 245-266 de Dukeminier, Property, Concise Edition 2E
2

Andy, Barb et Chet ont hérité de Blackacre, une ferme de 120 acres, à parts égales en
tant que locataires en commun. Peu de temps après, Andy a vendu sa participation d'un
tiers à Malls Inc. qui veut la totalité de la parcelle pour construire un nouveau centre
commercial. Barb ne veut pas que le terrain soit divisé, surtout pas par la vente, parce
qu'elle aime le terrain. Malls, Inc. demande au tribunal de procéder à un partage par
vente, en alléguant que la division du terrain en trois parties lui fera perdre de la valeur
et qu'il sera difficile d'équilibrer les parts. Lequel des énoncés suivants est vrai dans la
plupart des juridictions ?

(A) Barb peut annuler la vente de la part d'Andy à Malls, Inc. parce qu'Andy n'avait pas
son autorisation.

(B) Malls, Inc. ne peut demander le partage que si elle peut démontrer que le maintien
de la colocation est impossible ou irréalisable.

(C) Un tribunal ordonnera une vente à moins que Barb ne puisse prouver que le
partage en nature sera plus avantageux pour toutes les parties.

(D) Malls, Inc. peut obtenir le partage par la vente si elle peut démontrer que le
partage en nature est irréalisable ou qu'il en résultera un grand préjudice pour les
parties.

Raison d'être :
La bonne réponse estD. La réponseA est incorrecte car tout colocataire peut aliéner (vendre ou
donner) sa part de la succession à tout moment, sans autorisation, à moins qu'il n'existe une sorte
de restriction à l'aliénation de son intérêt. (S'il existait une telle restriction, elle devrait être
raisonnable pour être valable, par exemple en stipulant queA,B etC ne peuvent pas vendre
Blackacre de leur vivant). Dans le cas présent, il n'y a pas de restriction, et Andy a donc le droit
d'aliéner sans autorisation. Après la vente, Malls, Inc. devient locataire en commun avec Barb et
Chet. La réponseB est incorrecte car tout colocataire a le droit de procéder à un partage à tout
moment (à nouveau, tant qu'il n'y a pas de restriction à ses intérêts, ce qui n'est pas le cas en
l'espèce). Le colocataire n'a pas besoin de justifier d'un motif pour demander le partage. La seule
vraie question est donc de savoir si le partage se fera en nature (physique) ou par vente. En règle
générale, les tribunaux privilégient le partage en nature car la vente dépossède les parties de leurs
biens. Ainsi, le test habituel exige que la partie qui demande le partage par vente prouve que le
partage en nature est irréalisable ou qu'il causera un grand préjudice aux parties (par exemple, s'il
réduira considérablement la valeur du terrain à diviser en petits morceaux). La réponseC est donc
incorrecte car Barb n'a pas la charge de prouver que le partage en nature est plus avantageux. Au
lieu de cela, le partage en nature sera utilisé, à moins que Malls, Inc. ne prouve qu'il est impossible
à mettre en œuvre ou qu'il entraînera un préjudice important. Par conséquent, la réponseD est la
meilleure.
Lecture recommandée :
Pages 254-262 de Dukeminier, Property, Concise Edition 2E

3.
Ernst, Frieda et Gertrude achètent Blackacre, une maison de trois chambres à coucher,
et en prennent possession en tant que locataires en commun. Ils ont acheté Blackacre
en tant qu'investissement et prévoyaient de le louer. Cependant, Frieda et Gertrude ont
adoré la maison et ont décidé d'y emménager. Ernst veut que Frieda et Gertrude lui
paient un loyer pour vivre dans une maison qui lui appartient au tiers, après tout. Frieda
et Gertrude refusent catégoriquement et lui disent qu'il est libre d'emménager avec elles
s'il le souhaite. Lequel des éléments suivants est vrai ?
(A) Frieda et Gertrude doivent à Ernst les deux tiers de la valeur locative de la maison.

(B) Frieda et Gertrude doivent à Ernst un tiers de la valeur locative de la maison.

(C) Frieda et Gertrude ne doivent pas de loyer à Ernst, mais la valeur peut être
prise en compte pour déterminer la responsabilité des frais financiers tels que
l'assurance dans le cadre d'une action comptable.

(D) Si les trois étaient des locataires conjoints au lieu de locataires en commun, Frieda
et Gertrude devraient payer un loyer à Ernst.
Raison d'être :
La bonne réponse estC. Tout co-locataire peut posséder l'ensemble du bien sans être tenu de payer
un loyer à l'autre co-locataire. Chaque colocataire a le droit de posséder le bien, de sorte que la
théorie veut qu'il n'y ait pas de responsabilité, même si cela empêche le bien d'être utilisé pour un
revenu productif. La seule exception est l'éviction, lorsqu'un colocataire est empêché d'exercer son
droit de possession par les autres colocataires. Dans le cas présent, Frieda et Gertrude ont
explicitement invité Ernst à partager la possession avec elles, de sorte qu'il n'y a pas lieu de
l'évincer. Par conséquent, les réponsesA etB sont toutes deux incorrectes. En cas d'éviction, Ernst
aurait reçu un tiers de la valeur locative, et non deux tiers. La réponseD est incorrecte, car il n'y a
pas de différence entre une location conjointe et une location en commun en ce qui concerne les
droits et les responsabilités des colocataires. La réponseC est correcte car, même si Ernst ne peut
pas exiger des colocataires en possession de payer un loyer, la valeur de leur occupation peut être
prise en compte pour déterminer la responsabilité des frais financiers. Puisqu'ils tirent la valeur de la
maison, il est logique qu'ils paient l'assurance, par exemple.
Lecture recommandée :
Pages 262-266 de Dukeminier, Property, Concise Edition 2E

4.
Gramps voulait donner Blackacre à ses deux petits-enfants, Nick et Nora. Le lundi, il a
remis à Nick un acte de propriété de Blackacre qui stipule que "Gramps transmet à Nick
un demi-intérêt dans Blackacre". Le mardi, il a remis un acte à Nora, qui stipule que
"Gramps transmet un demi-intérêt dans Blackacre à Nora, en indivision avec Nick". Nick
est décédé peu de temps après, laissant un testament dans lequel il léguait tous ses
biens à sa femme, Wanda. Lequel des éléments suivants est vrai ?

(A) Wanda ne prend rien, en raison du droit de survie de Nora.

(B) Wanda prend un demi-intérêt dans Blackacre en tant que locataire en


commun avec Nora.

(C) Wanda ne prend rien, car Gramps ne peut pas transmettre Blackacre de cette
manière.

(D) Wanda ne prend rien, car Gramps voulait que seuls ses petits-enfants en profitent.
Raison d'être :
La bonne réponse estB. Gramps avait peut-être l'intention de créer une copropriété avec Nick et
Nora, mais il n'y est pas parvenu. Il n'a pas transmis à Nick et à Nora en même temps avec le même
instrument, manquant ainsi à deux des unités de titre requises. En outre, l'acte de cession à Nick ne
mentionne même pas l'existence d'une copropriété. Ainsi, lorsqu'il a cédé un demi-intérêt à Nick, il
en est résulté un bail en commun entre Gramps et Nick. Il a ensuite cédé sa part restante à Nora,
créant ainsi un bail commun entre Nick et Nora, malgré les termes de la cession. À la mort de Nick,
ses intérêts iraient donc à Wanda en vertu de son testament, ce qui donnerait lieu à une location en
commun entre Nora et Wanda. La réponseA est donc incorrecte ; il n'y a pas de droit de survie dans
une tenancy in common. La réponseC est incorrecte parce que Grand-père a réussi à transmettre
ses intérêts, mais pas en joint tenancy. La réponseD ne peut pas être correcte ; même si Grand-
père voulait que seuls ses petits-enfants en bénéficient, il n'a pas limité son transfert à un domaine
viager ou d'une autre manière. Ils ont donc pris chacun un demi-intérêt en fief simple absolu, qui est
cessible, héritable et aliénable. Certes, en common law, il aurait dû dire "à Nick et à ses héritiers",
mais cette formulation n'est plus requise et nous supposons que Nick était censé avoir son demi-
intérêt en fief simple absolu.
Lecture recommandée :
Pages 245-247 de Dukeminier, Property, Concise Edition 2E

5.
En 1990, il a acheté Blackacre, un ranch de 300 acres dans le Tennessee, et y a vécu
pendant de nombreuses années. En 2010, il a épousé W, mais ne lui a pas transmis
d'intérêts dans Blackacre. Le couple vivait dans un appartement loué en ville tandis que
le ranch était utilisé pour l'élevage de chevaux. Quelques années plus tard, le couple a
décidé de prendre sa retraite au Texas. Ils ont acheté un camping-car à Houston et y
ont vécu. Le Texas est un État de communauté de biens, ce qui n'est pas le cas du
Tennessee. Peu après, H a vendu Blackacre et a placé le produit de la vente (1 million
de dollars) sur un compte d'actions à son seul nom. H est ensuite décédé, domicilié au
Texas. Son testament laisse l'ensemble de ses biens à D, sa fille issue d'un précédent
mariage. Le véhicule de plaisance valait 200 000 dollars et était immatriculé au nom de
H. Le Tennessee dispose d'une loi sur la part élective, qui donne aux conjoints
survivants un tiers des droits sur la succession du conjoint décédé, alors que le Texas
n'en dispose pas. Si l'on fait abstraction des questions relatives à la propriété et au
douaire, laquelle des affirmations suivantes est vraie ?

(A) Lorsque H et W ont déménagé au Texas, W a automatiquement eu droit à un demi-


intérêt dans le ranch Blackacre en tant que propriété communautaire et devrait donc
recevoir la moitié du compte d'actions.

(B) En raison de la communauté de biens, W était copropriétaire du véhicule récréatif,


malgré son titre de propriété, et en était donc l'unique propriétaire au décès de H.

(C) W pourrait utiliser la disposition du Tennessee relative à la part élective pour obtenir
une part d'un tiers de tous les biens de la succession de H, en dépit du testament.

(D) W possède la moitié du véhicule de plaisance mais aucun compte d'actions.

Raison d'être :
La bonne réponse estD. Il s'agit du problème des "couples migrateurs". Lorsque le couple était
domicilié dans le Tennessee, W était protégé par la disposition relative à la part élective de l'État. Si
H était décédée alors qu'ils vivaient là, elle aurait eu droit à une part d'un tiers de Blackacre.
Cependant, lorsque le couple a déménagé au Texas, elle a perdu cette protection. Si Blackacre
n'avait pas été vendu, la réponse serait probablement différente car la loi du lieu de situation des
biens immobiliers (en l'occurrence le Tennessee) régit généralement la disposition des biens
immobiliers au moment du décès. En ce qui concerne les biens restants, la loi du Texas régira la
succession car c'est là que H était domicilié au moment du décès. Le compte d'actions ne peut pas
être considéré comme un bien commun car il ne provient pas des revenus de l'un ou l'autre des
époux pendant le mariage. H le possédait avant son mariage. Par conséquent, le compte d'actions
est également un bien distinct. Toutefois, le véhicule de plaisance a été utilisé pendant le mariage et
doit être considéré comme un bien commun, quel que soit son titre de propriété. La réponseA est
donc incorrecte : les biens ne sont pas automatiquement convertis en biens communs lorsqu'un
couple déménage dans un État où les biens sont communs. La réponseB est erronée car la
communauté de biens ne fait pas de W un co-locataire du véhicule de plaisance ; elle ne lui donne
qu'un demi intérêt dans le véhicule. Le couple aurait pu le titrer en copropriété, mais il ne l'a pas fait.
La réponseC est également erronée. La disposition du Tennessee relative à la part élective ne
s'applique pas parce que H est décédé domicilié au Texas.
Lecture recommandée :
Pages 266-267 de Dukeminier, Property, Concise Edition 2E

6.
Kim et Kanye étaient mariés et possédaient une maison de vacances appelée
Greenacre en tant que locataires à part entière. Un soir, Kim a trop bu et a eu un
accident. Elle a percuté une voiture conduite par Talia, la blessant grièvement. Talia a
intenté un procès et a obtenu un jugement de 2 000 000 $ contre Kim. Comme Kim
n'avait pas d'assurance suffisante pour payer ce montant, Talia veut exécuter le
jugement contre Greenacre. Lequel des énoncés suivants est vrai dans la plupart des
juridictions américaines qui autorisent la location par l'intégralité (tenancy by the
entirety) ?

(A) Talia devrait être en mesure d'obtenir la moitié des intérêts de Kim dans Greenacre,
car les jugements délictuels constituent généralement une exception à la règle
interdisant la séparation unilatérale.

(B) Talia ne pourra pas du tout saisir Greenacre, car les biens détenus en tenancy
by the entirety ne sont pas soumis aux dettes d'un seul conjoint.

(C) Talia ne sera pas en mesure d'atteindre Greenacre maintenant, mais elle peut créer
un privilège sur la propriété, qu'elle pourra faire valoir lors de la vente de la propriété.

(D) Talia devrait pouvoir accéder à la totalité de Greenacre, car Kim avait droit à la
possession de l'ensemble.
Raison d'être :
La bonne réponse estB. Les biens détenus en tenancy by the entirety ne peuvent être atteints par
les créanciers d'un seul conjoint. VoirSawada c. Endo ouRBS Citizens c. Ouhrabka. La plupart des
juridictions ne prévoient pas d'exception pour les jugements délictuels, lalettre A est donc
incorrecte. La réponseC n'est pas la règle dans la plupart des juridictions qui utilisent le TBE, à
l'exception des privilèges fiscaux fédéraux. Kim avait certainement droit à la possession de
l'ensemble, comme l'indique la réponseD, mais l'idée même du "tenancy by entirety" est d'empêcher
l'un des époux de grever unilatéralement le bien, de sorte que le droit à la possession n'est pas
pertinent.

Chapitre 6

Kaya loue un loft dans le centre-ville d'Arcadia City à Cliff, étudiant en droit, pour une
durée de trois ans et un loyer mensuel de 2 500 dollars. Un an plus tard, après avoir
obtenu un stage en dehors de l'État, Cliff "sous-loue, transfère et cède" le bien à son
camarade de classe Thomas pour "une période d'un an à compter de la date
d'aujourd'hui". Par la suite, ni Thomas ni Cliff ne paient de loyer à Kaya. Kaya a-t-elle
des droits à l'encontre de Cliff ?

(A) Oui, Kaya a des droits contre Cliff parce que le transfert à Thomas est une cession
et que Thomas n'a pas payé de loyer.

(B) Oui, Kaya a des droits contre Cliff parce que le transfert à Thomas est une
sous-location et que Thomas n'a pas payé de loyer.

(C) Non, Kaya n'a pas de droits à l'encontre de Cliff car le transfert à Thomas est une
cession, et Thomas est redevable à Kaya de la totalité du loyer.

(D) Non, Kaya n'a pas de droits à l'encontre de Cliff car le transfert à Thomas est une
sous-location et Thomas est redevable à Kaya de la totalité du loyer.
Raison d'être :
La bonne réponse est(B). Cliff a un bail de trois ans et n'a transféré qu'une année à Thomas. Étant
donné qu'il n'a pas transféré l'intégralité de ses droits - notons qu'il reste à Cliff un an de bail après
le départ de Thomas - ce transfert est qualifié de sous-location. Étant donné que le bailleur et le
locataire initial restent liés par le lien de succession et le lien contractuel, le locataire reste redevable
du loyer. Par conséquent, les points(A) et(C) sont incorrects. Lepoint (D) est également incorrect
parce qu'il confond la partie d'un contrat de sous-location qui est légalement responsable du loyer
envers le propriétaire. La partie qui doit payer le loyer dans une sous-location est toujours le
locataire initial (le sous-bailleur), et non le sous-locataire. VoirErnst v. Conditt, 390 S.W.2d 703
(Tenn. 1964).
Lecture recommandée :
Pages 294-308 de Dukeminier, Property, Concise Edition 2E
2

Lance est propriétaire d'un immeuble d'habitation. Tracy voulait louer l'appartement 2
de Lance, mais elle a remarqué qu'il présentait plusieurs problèmes. L'évier était
bouché, il y avait des fils apparents là où un luminaire se détachait, et une forte odeur
de moisissure se dégageait du placard, où le plafond semblait présenter des signes de
dégâts d'eau persistants. Lance a reconnu les problèmes, mais a déclaré qu'il n'avait ni
le temps ni l'argent pour réparer l'appartement : "Je vais vous dire, a dit Lance à Tracy,
normalement cet appartement se vend 600 dollars par mois, mais vu son état, je peux
vous le louer pour 400 dollars. Tracy a signé le bail, mais plusieurs mois plus tard, elle a
cessé de payer le loyer, affirmant que les odeurs de moisissure étaient insupportables
et que les autres défauts rendaient l'endroit dangereux et insalubre. Lequel des
éléments suivants est vrai ?

(A) Tracy ne peut pas invoquer une violation de la garantie implicite d'habitabilité, car
elle n'a pas déménagé et, par conséquent, il n'y a pas d'expulsion implicite.

(B) Tracy ne peut pas invoquer une violation de la garantie implicite d'habitabilité, car
elle y a renoncé dans le bail en échange d'un prix de location moins élevé.

(C) Tracy peut invoquer une violation de la garantie implicite d'habitabilité, mais
seulement si certains des défauts dont elle se plaint sont contraires au code du
logement.

(D) Tracy a de bonnes raisons d'invoquer une violation de la garantie implicite


d'habitabilité, ce qui peut être utilisé comme moyen de défense si Lance intente
une action pour loyers impayés.

Raison d'être :
La bonne réponse estD. La garantie implicite d'habitabilité (GIA) protège les locataires résidentiels
contre les conditions qui ont un impact significatif sur la sécurité et la santé. La réponseA n'est pas
correcte car, contrairement à la clause de jouissance paisible, l'IWH n'oblige pas le locataire à
déménager et à réclamer une expulsion implicite. La réponseB est également incorrecte car les
tribunaux n'autorisent pas la renonciation à l'IWH, même si le locataire obtient un loyer plus bas en
échange d'une tentative de renonciation. Certains Residential Landlord Tenant Acts autorisent des
dérogations dans des circonstances limitées, par exemple pour les maisons unifamiliales, mais pas
dans un cas comme celui-ci. La réponseC est trop limitée ; dans la plupart des juridictions, même si
les violations ne sont pas couvertes par le code du logement, dès lors que les conditions rendent les
locaux dangereux ou insalubres, l'IWH est violé. Dans ce cas, la moisissure, les fils électriques
exposés et l'évier bouché semblent suffisants pour prouver que l'IWH n'a pas été respecté.
Lecture recommandée :
Pages 334-344 de Dukeminier, Property, Concise Edition 2E
3

Lana a loué Blackacre à Tad pour une période de dix ans, du 1er janvier 2006 au 31
décembre 2015, au prix de 5 000 $ par mois. En 2008, Tad a conclu un accord avec
Fiona pour lui transférer ce droit de bail pour le reste de la durée de la location. Fiona a
accepté d'assumer toutes les responsabilités liées au bail et de payer le loyer mensuel
de 5 000 dollars directement à Lana. Lana a approuvé l'accord mais n'a pas adhéré au
contrat. Deux ans plus tard, en 2010, Fiona a transféré le mandat restant à Sal. Là
encore, Lana approuve l'accord et Sal accepte d'assumer toutes les responsabilités du
bail et de payer le loyer mensuel de 5 000 dollars directement à Lana. Un an plus tard,
en 2011, Sal a cessé de payer le loyer et Lana veut intenter une action en justice pour
récupérer les loyers dus. Laquelle de ces trois parties peut-elle poursuivre ?
(A) Tad et Sal

(B) Tad, Fiona et Sal

(C) Sal uniquement

(D) Tad uniquement


Raison d'être :
La bonne réponse estA. Les deux transferts à Fiona et Sal sont des cessions car ils portent sur la
totalité de la durée restante et transfèrent tous les droits et responsabilités découlant du bail
principal. Par conséquent, lorsque Fiona prend la relève en tant que cessionnaire, Lana est toujours
en relation contractuelle avec Tad (elle sera toujours en relation contractuelle avec Tad à moins
qu'elle ne le libère de ses obligations contractuelles). (Elle sera toujours en relation contractuelle
avec Tad à moins qu'elle ne le libère de ses obligations contractuelles.)C est donc incorrect. Lana
est alors en relation de succession avec Fiona, car cette dernière a repris la succession de Tad.
Lorsque Fiona transfère la succession à Sal, une autre cession, Sal a un lien de connexité avec
Lana etD est donc incorrect. À ce moment-là, Fiona n'a plus de lien patrimonial et n'a jamais eu de
lien contractuel. B est donc incorrect. Lana peut poursuivre Tad au titre de la connexité contractuelle
et Sal au titre de la connexité patrimoniale.
Lecture recommandée :
Pages 294-300 de Dukeminier, Property, Concise Edition 2E
4

Lucky, le propriétaire, et Tessa, la locataire, concluent le 1er novembre 2018 un bail


d'une durée d'un mois qui prévoit un loyer de 1 200 $ par mois. Tessa a toujours payé
son loyer à temps et en totalité, mais elle envisage maintenant de déménager chez ses
parents pour économiser de l'argent. À la date d'aujourd'hui (15 décembre), la date la
plus proche à partir de laquelle Tessa n'est plus responsable du loyer est la suivante :

(A) Le 31 décembre.

(B) Le 31 janvier.

(C) Le 1er janvier.

(D) Le 28 février.
Raison d'être :
La bonne réponse est(B) : un bail périodique est conclu pour une période déterminée, en
l'occurrence d'un mois à l'autre. Il n'y a pas de faits qui suggèrent que cet accord a créé une location
périodique d'année en année. Dans le cas d'une location périodique d'un mois à l'autre, chaque
partie peut résilier le contratmoyennant un préavis d'au moins un mois. Ici, Tessa décide de
déménager le 15 décembre. Elle doit donner un préavis d'au moins un mois (au moins 30 jours)
pour éviter d'être tenue responsable du loyer. Par conséquent, les points(A) et(C) doivent être
incorrects, car ces dates ne permettraient de respecter qu'un préavis de 16 ou 17 jours,
respectivement.(D) doit également être incorrect, car il n'existe pas de règle exigeant un préavis de
deux mois (ici, une période égale à deux fois la durée du bail) pour mettre fin à une location
périodique de mois en mois.
Lecture recommandée :
Pages 281-287 de Dukeminier, Property, Concise Edition 2E
5

Lora est propriétaire du Mayfair Mall et loue l'un des bâtiments aux restaurants
McBurgers pour une période de cinq ans. Plusieurs mois après le début du bail,
McBurgers a eu des difficultés à faire fonctionner correctement le siphon de sol, ce qui
a provoqué l'apparition d'eau sur le sol chaque fois que les employés mettaient plus
d'un gallon d'eau dans l'évier. Le bail indique que McBurgers a pris les locaux "en l'état"
et contient une disposition imposant à McBurgers l'obligation "d'effectuer toutes les
réparations nécessaires dans les locaux". Lequel des énoncés suivants est vrai dans la
plupart des juridictions ?

(A) La garantie implicite d'habitabilité a probablement été violée ici, car ce problème
touche à la santé et à la sécurité et ne peut pas faire l'objet d'une dérogation dans le
bail.

(B) McBurgers ne pourra pas invoquer une violation de la garantie implicite


d'habitabilité, car il n'a pas subi d'"éviction implicite".

(C) McBurgers ne pourra pas invoquer une violation de la clause de jouissance paisible,
car ce problème n'a rien à voir avec le bruit.

(D) McBurgers aurait plus de poids pour invoquer une violation de la clause de
jouissance paisible si la cause du problème d'évacuation des eaux se trouvait
dans une partie commune du centre commercial Mayfair, à l'extérieur du
restaurant.

Raison d'être :
D est la bonne réponse. Rappelez-vous que dans la plupart des juridictions, la garantie implicite
d'habitabilité ne s'applique qu'aux logements résidentiels, de sorte que les réponsesA etB peuvent
être écartées.B est également fausse parce que l'IWH n'exige pas d'éviction implicite. La loi accorde
généralement aux baux commerciaux une marge de manœuvre beaucoup plus grande pour
négocier les obligations de réparation, compte tenu de la diversité des situations et de l'égalité
relative du pouvoir de négociation. La clause de jouissance paisible s'applique aux locaux
commerciaux. Cependant, la réponseC est incorrecte car la jouissance "tranquille" ne se réfère pas
uniquement au bruit, mais plutôt à tout ce qui perturbe l'utilisation et la jouissance des locaux par le
locataire. L'interférence doit toutefois résulter d'une cause que le bailleur a causée ou à laquelle il a
l'obligation de remédier. Par conséquent, le pointD est correct. Dans ce cas, Lora n'est pas
responsable de l'état des locaux loués en vertu du bail. Néanmoins, les propriétaires sont
responsables de l'entretien et de la réparation des parties communes, de sorte que si le problème
provient de l'extérieur des locaux loués, McBurgers devrait avoir de bonnes chances de s'en sortir.
Lecture recommandée :
Pages 334-344 de Dukeminier, Property, Concise Edition 2E
6
Le 1er juillet, le locataire Tom a loué un appartement au propriétaire Lu au mois pour 1
000 $/mois et pour une période minimale de dix mois. Les deux parties se sont mises
d'accord par une poignée de main et un accord verbal plutôt que par écrit. Lorsque Tom
a emménagé, l'appartement était en bon état. Le 1er novembre, cependant, les toilettes
de Tom ont cessé de fonctionner et l'évier de la cuisine s'est bouché. Tom a appelé Lu
immédiatement et Lu a dit qu'il allait "s'en occuper". Trois jours plus tard, cependant,
rien n'avait été fait. Tom a donc informé Lu par écrit qu'il souhaitait que des mesures
soient prises rapidement pour remédier à ces défauts. En réponse, Lu est venu et a fait
des recherches sur les toilettes et l'évier, mais n'a pas réussi à les réparer. Il a dit qu'il
devait "appeler mon ami Brian". Il peut réparer ce genre de choses". Tom ne pouvait
pas cuisiner car il ne pouvait pas utiliser l'évier et devait utiliser les toilettes de son
voisin ou descendre dans le hall d'entrée si son voisin n'était pas là. Le 1er décembre,
Lu n'ayant toujours pas réglé les problèmes, Tom n'a pas payé son loyer. Le mois de
janvier est arrivé et les problèmes n'étaient toujours pas résolus, si bien que Tom n'a
pas payé le loyer de janvier non plus. Lu a finalement réglé le problème le 15 janvier et
veut maintenant que Tom paie l'arriéré de loyer. Parmi les doctrines suivantes, laquelle
est la meilleure défense de Tom face à cette affirmation ?

(A) Pacte de jouissance paisible

(B) Garantie implicite d'habitabilité

(C) Réparation et déduction

(D) Statut des fraudes


Raison d'être :
La bonne réponse estB. La garantie implicite d'habitabilité impose au bailleur de maintenir les locaux
dans un état convenable et habitable. Il y a violation lorsqu'il y a interférence matérielle avec les
conditions d'habitabilité, ce qui semble être le cas ici. En outre, le locataire doit informer le
propriétaire du défaut et lui accorder un délai raisonnable pour y remédier, ce que Tom a fait en
l'espèce. Par conséquent, l'IWH serait une défense viable. La réponseA est incorrecte car la clause
de jouissance paisible ne s'applique pas. Tom n'a pas quitté l'appartement (c'est-à-dire qu'il n'a pas
été expulsé de manière constructive) et ne peut donc pas utiliser cette doctrine. Tom aurait pu
utiliser la réparation et la déduction pour ces infractions, mais il aurait dû engager quelqu'un pour
réparer lui-même les défauts et déduire ensuite les coûts de réparation, ce qu'il n'a pas fait. La
réponseC est donc incorrecte. La loi sur les fraudes exige que seuls les baux de plus d'un an soient
rédigés par écrit pour être applicables. Par conséquent, le Statute of Frauds n'aidera pas Tom dans
ce cas. (Vous pouvez également noter que des exceptions telles que l'exécution partielle pourraient
rendre le bail exécutoire même si le Statute of Frauds s'appliquait). La réponseD est incorrecte.
Lecture recommandée :
Pages 334-344 de Dukeminier, Property, Concise Edition 2E
7

Margie's Muffins a signé un bail de cinq ans pour un espace commercial de boulangerie
à Arcadia City appartenant à Langham Realty Co. Après quatre ans, Margie a décidé
de prendre une retraite anticipée et a prévu de sous-louer l'espace à Christine's
Croissants, un boulanger concurrent de la ville. Margie a demandé à son avocat de
rédiger un contrat de sous-location d'un an pour couvrir l'arrangement. L'accord
comprenait une disposition selon laquelle Christine's payait le loyer directement à
Langham, que "Margie's Muffins est par la présente libérée de toutes les obligations
découlant du contrat de bail initial" et que "le sous-locataire accepte d'assumer tous les
devoirs et obligations découlant du contrat de bail initial". Margie's, Christine's et
Langham ont signé l'accord. Après six mois dans les locaux, Christine cesse de payer
le loyer. Langham poursuit Margie's pour non-paiement du loyer.
Langham a-t-il des chances de réussir ?

(A) Oui, car le locataire initial est responsable du loyer si le sous-locataire ne le paie
pas.

(B) Oui, car le locataire initial reste lié au propriétaire par un contrat.

(C) Non, car le propriétaire a consenti à l'arrangement.

(D) Non, car le locataire initial a cédé ses droits sur l'espace et n'est plus en
relation de succession ou de contrat avec le propriétaire.

Raison d'être :
La bonne réponse est(D). L'analyse porte sur la question de savoir si l'accord crée une cession ou
une sous-location. Étant donné que Margie n'a conservé aucun droit de réversion sur le bien (la
"sous-location" concerne la dernière année de la durée initiale de cinq ans du bail de Margie) et que
le deuxième accord comprenait une libération de toutes les obligations pour Margie, cet accord doit
très probablement être considéré comme une cession.Voir Ernst c. Conditt.(A) est incorrect parce
qu'il ne fait pas la distinction entre une sous-location et une cession. Dans le cas d'une cession, le
lien de propriété et le contrat entre le locataire initial et le propriétaire sont détruits.(B) est incorrecte
car le second contrat comprend une clause libérant le locataire initial des engagements du contrat
initial.(C) est incorrect car le consentement du propriétaire n'est pas déterminant pour la question de
la responsabilité du locataire.
Lecture recommandée :
Pages 294-308 de Dukeminier, Property, Concise Edition 2E
8

Le 1er juillet, le locataire Tom a loué un appartement au propriétaire Lu au mois pour 1


000 $/mois et pour une période minimale de dix mois. Les deux parties se sont mises
d'accord par une poignée de main et un accord verbal plutôt que par écrit. Lorsque Tom
a emménagé, l'appartement était en bon état. Le 1er novembre, cependant, les toilettes
de Tom ont cessé de fonctionner et l'évier de la cuisine s'est bouché. Tom a appelé Lu
immédiatement et Lu a dit qu'il allait "s'en occuper". Trois jours plus tard, cependant,
rien n'avait été fait. Tom a donc informé Lu par écrit qu'il souhaitait que des mesures
soient prises rapidement pour remédier à ces défauts. En réponse, Lu est venu et a fait
des recherches sur les toilettes et l'évier, mais n'a pas réussi à les réparer. Il a dit qu'il
devait "appeler mon ami Brian". Il peut réparer ce genre de choses". Tom ne pouvait
pas cuisiner car il ne pouvait pas utiliser l'évier et devait utiliser les toilettes de son
voisin ou descendre dans le hall d'entrée si son voisin n'était pas là. Le 1er décembre,
Lu n'ayant toujours pas réglé les problèmes, Tom n'a pas payé son loyer. Le mois de
janvier est arrivé et les problèmes n'étaient toujours pas résolus, si bien que Tom n'a
pas payé le loyer de janvier non plus. Lu a finalement réglé le problème le 15 janvier et
veut maintenant que Tom paie l'arriéré de loyer. En supposant que le tribunal juge le
bail valable, quel est le montant le plus probable que Tom devra payer à Lu pour
l'arriéré de loyer ?

(A) La valeur des locaux dans leur état défectueux

(B) Tom ne devrait pas avoir à payer d'arriérés de loyer à Lu.

(C) Le loyer convenu moins le montant des réparations nécessaires

(D) Le montant total du loyer moins les dommages-intérêts pour souffrances et douleurs
Raison d'être :
La bonne réponse estA. Bien que les tribunaux utilisent une variété de mesures de dommages pour
une violation de la garantie implicite d'habitabilité, la différence entre le loyer convenu et la valeur
des locaux dans leur état défectueux est l'une des plus courantes. Étant donné que les locaux
avaient encore une certaine valeur pour Tom, il ne sera probablement pas complètement libéré de
son obligation de location, de sorte queB est incorrect.C est incorrect parce qu'aucune juridiction
n'utilise cette formule ; il est certain que si Tom avait utilisé le recours en réparation et en déduction,
cela aurait été approprié, mais il ne l'a pas fait. En outre, les juridictions n'utilisent généralement pas
le calcul de la douleur et de la souffrance, préférant considérer la valeur diminuée des locaux
comme la mesure appropriée du préjudice subi par Tom. La réponseD est donc incorrecte.
Lecture recommandée :
Pages 341-342 de Dukeminier, Property, Concise Edition 2E
9

Paul possédait une propriété, Wedgewood, à proximité de l'université. Son testament se


lit comme suit : "Je lègue Wedgewood à ma fille Amelia pour 50 ans : "Je lègue
Wedgewood à ma fille Amelia pour 50 ans, puis à l'université". Lequel des éléments
suivants est vrai ?

(A) L'université a un droit exécutoire.

(B) L'université dispose d'un reliquat éventuel.

(C) Amelia a une durée de vie de plusieurs années.

(D) Le droit de l'Université est nul en vertu de la règle contre les perpétuités.
Raison d'être :
La bonne réponse estC. Laréponse A est incorrecte parce que University a un droit futur sur un
tiers après l'expiration naturelle d'un délai d'un an, ce qui s'appelle un droit résiduel et non un droit
exécutoire. La réponseB est incorrecte car le reliquat de l'Université est acquis plutôt qu'éventuel
(pas de condition suspensive, nous savons qui est le preneur). La réponseD est également
incorrecte car la règle contre les perpétuités ne s'applique pas à l'un ou l'autre des intérêts ici -
rappelez-vous qu'elle ne s'applique qu'aux reliquats conditionnels et aux intérêts exécutifs ;
University a un reliquat VESTE, donc la règle ne s'applique pas.
Lecture recommandée :
Page 281 de Dukeminier, Property, Concise Edition 2E
10

Les frères et sœurs Landon et Mica ont hérité de la petite entreprise immobilière de leur
mère à Arcadia City. Le portefeuille de la fratrie comprend quatre maisons individuelles
et deux appartements qui sont souvent loués comme locations de vacances. Pour
tenter de faire louer l'une des maisons, Landon a publié un message sur les médias
sociaux :
''Maison unifamiliale à louer à des célibataires ou à des couples sans enfants''.
Vous êtes avocat au bureau du procureur général et vous remarquez le message en
faisant défiler votre propre fil d'actualité. L'affichage est-il contraire à la loi fédérale sur
le logement équitable ?

(A) Oui, car la loi sur les droits civils de 1866 interdit la publicité discriminatoire.

(B) Oui, car la loi sur le logement équitable (Fair Housing Act) interdit les
affichages de ce type.

(C) Non, car la loi sur les droits civils n'est pas applicable au logement.

(D) Non, en raison des dérogations législatives aux lois fédérales sur le logement
équitable aux États-Unis.
Raison d'être :
La bonne réponse est(B). Les messages publiés sur les médias sociaux constituent une publicité,
telle que définie par la loi sur le logement équitable (Fair Housing Act), 42 U.S.C. 3601et seq. La
formulation "aux célibataires ou aux couples sans enfants" exprime une préférence claire pour
exclure les familles avec enfants de la propriété, ce qui constitue une forme de discrimination fondée
sur le statut familial. (Laréponse A est incorrecte car, bien que l'ARC interdise la discrimination, ses
protections ne s'étendent pas aux annonces. (La réponseC est incorrecte car la loi sur les droits
civils s'applique au logement. (D) est incorrecte car les annonces discriminatoires ne sont soumises
à aucune des exemptions de la FHA (ainsi, les faits concernant le portefeuille des frères et sœurs ne
sont pas pertinents et n'entraînent pas d'exemption).
Lecture recommandée :
Pages 288-293 de Dukeminier, Property, Concise Edition 2E
11

Tabatha a signé un bail pour louer un appartement à Linda sur une base
hebdomadaire, la location commençant le lundi 2 janvier. Au bout de neuf semaines,
elle a décidé de retourner vivre chez sa mère. Elle a payé le loyer le lundi 27 février
pour la semaine suivante. Le vendredi 3 mars, elle a annoncé à Linda qu'elle
déménagerait au cours du week-end et qu'elle résilierait le bail. Elle a remis à Linda une
note écrite à cet effet. Linda a répondu qu'elle devrait également payer pour la semaine
suivante, jusqu'au dimanche 12 mars. En l'absence d'autorité statutaire sur la question,
laquelle des affirmations suivantes est vraie ?

(A) Tabatha doit à Linda un loyer pour toute la semaine prochaine, jusqu'au
dimanche 12 mars.

(B) Tabatha doit à Linda un loyer pour une partie de la semaine prochaine, jusqu'au
vendredi 10 mars.

(C) Tabatha ne doit pas de loyer supplémentaire à Linda ; elle est payée jusqu'à cette
semaine et a donné à Linda un préavis suffisant.

(D) Tabatha ne doit pas de loyer supplémentaire à Linda, car le bail n'a pas spécifié le
préavis requis.
Raison d'être :
La bonne réponse estA. Dans le cas d'une location périodique, ce qui est clairement le cas ici, la
common law exige un préavis égal à la durée de la période - dans ce cas, une semaine. Laréponse
C est donc incorrecte. En outre, le droit commun exige que la résiliation intervienne à la fin de la
période - dans ce cas, le dimanche. Par conséquent, à ce stade, la résiliation de Linda ne peut
prendre effet qu'à la fin de la semaine suivante, de sorte qu'elle doit le loyer pour toute cette
semaine. La réponseB est donc incorrecte. La réponseD est incorrecte ; il n'est pas nécessaire que
le bail contienne des dispositions relatives à la résiliation et au préavis.
Lecture recommandée :
Pages 281-282 de Dukeminier, Property, Concise Edition 2E
12

La serrure de la porte d'entrée de la boutique de vêtements pour enfants de Hannah ne


fonctionne pas correctement depuis qu'elle a loué les locaux il y a treize (13) ans. Elle a
informé le propriétaire de la rupture de la serrure au cours de la première année de son
bail, mais il lui a dit : "Ce n'est pas grave. N'hésitez pas à le réparer vous-même".
Hannah est frustrée par l'attitude désinvolte du propriétaire et a décidé de quitter les
lieux ce mois-ci. Si elle quitte le logement ce mois-ci, devra-t-elle payer un loyer
supplémentaire ?

(A) Oui, car le propriétaire n'a manqué à aucun de ses devoirs ou obligations.

(B) Oui, car le propriétaire a violé le pacte de jouissance paisible.

(C) Non, car dans le cadre d'une action pour loyer impayé, elle peut invoquer la défense
d'expulsion implicite en raison de la serrure cassée.

(D) Non, car le propriétaire n'a pas respecté la garantie implicite d'habitabilité.
Raison d'être :
La bonne réponse est(A). Bien qu'un propriétaire résidentiel soit généralement responsable des
mesures de sécurité de base, telles que les serrures de porte, le défaut ici est mineur et concerne
un bail commercial. La boutique est restée ouverte pendant treize ans, même avec la serrure
cassée, ce qui suggère que Hannah a identifié une méthode alternative pour sécuriser les lieux.(B)
est incorrect car sa logique est incohérente. Si le propriétaire a violé l'engagement de jouissance
paisible, Hannah peut être libérée de son obligation de payer le loyer.(C) est incorrect car il n'y a
expulsion implicite que lorsque le locataire s'est vu refuser la jouissance effective des locaux. En
l'espèce, Hannah est restée en possession de son magasin et a continué à l'exploiter pendant 13
ans avec la serrure cassée. Elle n'a pas été privée de la jouissance des locaux. (Lepoint D) est
incorrect pour deux autres raisons : Premièrement, parce que la garantie implicite ne s'applique
généralement pas aux baux commerciaux. Deuxièmement, même si cette juridiction reconnaît la
garantie implicite d'habitabilité dans les baux commerciaux, la possession de l'espace par Hannah
pendant 13 ans suggère que les locaux sont adaptés aux fins pour lesquelles ils ont été loués.
Lecture recommandée :
Pages 322-344 de Dukeminier, Property, Concise Edition 2E
13

Torry a été déployé en Afghanistan et a décidé d'abandonner son appartement, mais il


avait besoin d'un endroit pour stocker sa précieuse Corvette pendant son absence. Lisa
a écrit à Torry pour lui proposer de lui louer son garage pour un loyer de 300 dollars par
mois "pendant la durée de votre déploiement". Signé, Lisa Landlord". Torry a
immédiatement appelé Lisa et a accepté l'offre. Après la première année, l'unité de
Torry a été jugée cruciale pour une opération en cours et son déploiement a été
prolongé de douze mois. Lisa n'est pas contente car elle envisage maintenant de
vendre la maison et veut résilier le bail de Torry et sortir la voiture du garage. Torry
refuse, disant qu'il ne peut pas s'occuper de cela depuis une zone de guerre. Si Lisa
tente de résilier le bail, lequel des éléments suivants constituera le plus gros problème
pour Torry, qui devra s'y tenir ?

(A) Un tribunal peut interpréter le bail comme une location à volonté.

(B) Une juridiction peut interpréter le bail comme une durée de plusieurs années.

(C) Un bail est automatiquement résilié en cas de vente du fief simple sous-jacent.

(D) Le statut des fraudes rendra le bail inapplicable.


Raison d'être :
La bonne réponse estA. Un doute subsiste quant à la catégorie de bail dont il s'agit. Il pourrait être
interprété comme un terme d'années, qui dure pendant une période donnée - dans ce cas, jusqu'à la
fin de la guerre. Cela favoriserait Torry car Lisa ne pourrait pas résilier le bail avant la fin de cette
période et rendraitB incorrect. Toutefois, comme la guerre n'a pas de date de fin déterminée, un
tribunal pourrait l'interpréter comme un contrat de location à volonté, qui peut généralement être
résilié par l'une ou l'autre des parties. Si c'est le cas, Lisa peut résilier le contrat (généralement avec
un préavis d'un mois).C est incorrect car l'acheteur d'un bien immobilier est lié par tous les baux
antérieurs dont il a connaissance.D est incorrect car la lettre de Lisa est une note suffisante de
l'accord en vertu de la loi sur les fraudes, signée par la partie qui doit être liée. Le fait que Torry n'ait
pas signé n'est pas pertinent, à moins que Lisa n'essaie d'obliger Torry à respecter les conditions du
contrat.
Chapitre 7

A a accepté de vendre Blackacre à B pour 200 000 dollars. Ils ont conclu un contrat de
vente exécutoire le 1er janvier 2016, avec une date de clôture fixée au 1er mars 2016,
sous réserve des conditions habituelles de financement et de titre de propriété
négociable. Cependant, le 15 février, A est décédé. Le testament de A léguait tous ses
biens personnels à sa fille, D, et léguait tous ses biens immobiliers à son fils, S. Lequel
des points suivants est vrai ?

(A) Si la juridiction applique la conversion équitable, S aurait le droit le plus fort au


produit de la vente.

(B) Si la juridiction applique la conversion équitable, D aurait le droit le plus fort


au produit de la vente.

(C) Que la juridiction applique ou non la conversion équitable, cette vente peut être
annulée par l'exécuteur testamentaire parce que A est décédé avant la conclusion du
contrat et que le contrat était soumis à des aléas.

(D) La vente devrait avoir lieu, mais dans la plupart des juridictions, S et D devraient
partager le produit de la vente, selon le principe de l'étouffement.
Raison d'être :
La bonne réponse estB. Cette question vous demande d'envisager l'application de la conversion
équitable et, bien sûr, de comprendre la différence entre les biens meubles et les biens immeubles.
Si le tribunal appliquait la conversion équitable, il considérerait la vente comme achevée au moment
de la signature du contrat, de sorte que B serait le propriétaire équitable de Blackacre au 1er janvier.
À ce moment-là, A n'aurait qu'un droit équitable sur le produit de la vente, qui est un bien personnel.
Par conséquent, si la juridiction applique la conversion équitable, D, le légataire, aurait le droit le
plus fort. La réponseA est donc incorrecte car elle présente de manière erronée le résultat de
l'application de la conversion équitable. La réponseC est incorrecte car les contrats immobiliers
valides et exécutoires ne sont pas affectés par le décès de l'une des parties. En outre, même s'il y a
des imprévus, les tribunaux appliqueront la conversion équitable pour autant qu'il s'agisse
d'imprévus normaux. La réponseD est incorrecte car le testament précise clairement quel
bénéficiaire a droit à quel type de bien ; il n'y a aucune raison pour que le tribunal s'écarte de
l'intention du testateur.
Lecture recommandée :
Page 370 de Dukeminier, Property, Concise Edition 2E
2

Alf Landon a donné à la First Bank une hypothèque sur Blackacre en échange d'un prêt
de 50 000 dollars dans une juridiction de notification. La First Bank a enregistré
l'hypothèque, mais le bureau du recorder a mal indexé l'hypothèque sous Alf "London"
au lieu de "Landon". Un an plus tard, Alf a vendu la propriété à Betty par un acte de
garantie générale. Elle l'a immédiatement revendue à Carl en utilisant un acte de
garantie spécial. Carl l'a transféré quelques jours plus tard à Dana par acte de
renonciation. Quelques mois plus tard, Alf n'a pas remboursé le prêt et la First Bank a
entamé une procédure de saisie à l'encontre de Blackacre. Lequel des énoncés
suivants est le plus vraisemblablement vrai dans la plupart des États ?

(A) La First Bank ne peut pas faire valoir son hypothèque à l'encontre de Dana, car
l'hypothèque n'a pas été correctement enregistrée.

(B) L'hypothèque est toujours valable et Dana devrait pouvoir poursuivre Betty sur la
base des garanties contenues dans l'acte que Betty a donné à Carl.

(C) L'hypothèque est toujours valable et Dana devrait pouvoir poursuivre Alf sur la base
de la convention contre les charges.

(D) L'hypothèque est toujours valable et Dana devrait pouvoir poursuivre Alf sur
la base de la clause de jouissance paisible.

Raison d'être :
La bonne réponse estD. Dans cette question, vous devez vous demander si les garanties de titre
"courent avec le terrain" pour les propriétaires ultérieurs. Mais il faut d'abord se demander si
l'hypothèque de la First Bank est encore valable. Il a été enregistré en temps utile mais
incorrectement indexé. La plupart des juridictions considèrent que l'erreur d'indexation, si elle n'est
pas imputable à la personne qui enregistre, est toujours considérée comme une notification
d'enregistrement, même si elle rend l'acte difficile à découvrir dans le cadre d'une recherche
normale de titres. Dans ce cas, cela signifie que la First Bank gagne parce qu'elle a fait ce qu'elle
était censée faire - enregistrer l'hypothèque. La réponseA est donc incorrecte. Dana doit donc se
retourner contre quelqu'un pour obtenir réparation de son préjudice. Elle ne peut pas poursuivre Carl
sur la base des garanties parce qu'il n'a donné qu'un acte de renonciation, qui ne comporte aucune
garantie. Betty n'a donné qu'un acte de garantie spéciale, qui ne garantit que les défauts survenus
pendant que Betty était propriétaire de Blackacre. Ce défaut s'est produit avant qu'elle n'obtienne le
titre de propriété, de sorte que Dana ne peut pas poursuivre Betty. La réponseB est donc incorrecte.
Une action contre Alf est possible car Alf a donné à Betty un acte de garantie générale. La règle
générale est que les garanties de titre actuelles ne s'appliquent pas aux propriétaires ultérieurs,
alors que les garanties futures s'appliquent. Par conséquent, étant donné que le pacte contre les
charges est une garantie actuelle, la réponseC est incorrecte. La clause de jouissance paisible,
réponseD, est une garantie future et s'applique aux acquéreurs ultérieurs comme Dana. Il n'est violé
que lorsque la possession de Dana est perturbée par quelqu'un qui revendique un titre de propriété
supérieur, ce qui décrit précisément la saisie effectuée par la First Bank dans cette affaire. Par
conséquent, la réponseD est la plus susceptible d'être le résultat dans la plupart des États.
Lecture recommandée :
Pages 388-394 de Dukeminier, Property, Concise Edition 2E
3

Alf souhaite que sa fille Becky hérite de Blackacre à sa mort. Il rédige un acte
concernant Blackacre qui stipule que "Alf concède par la présente Blackacre à Becky, à
ses héritiers et à ses ayants droit pour toujours". Le jour de son anniversaire, il lui
montre l'acte et lui dit qu'il sera dans un tiroir de son bureau avec son testament qui
donne tous ses biens aux Odd Fellows, une organisation caritative. À la mort d'Alf, son
exécuteur testamentaire retrouve l'acte et le remet à Becky, qui s'empresse de
l'enregistrer. Becky est-elle propriétaire de Blackacre ?

(A) Non, car il n'y a pas eu de contrepartie à l'acte.

(B) Non, car le testament prévaut sur tout transfert de propriété par donation, sauf si
l'acte de donation est enregistré avant le décès du constituant.

(C) Oui, car le don a eu lieu avant la mort d'Alf, même s'il n'a été enregistré que plus
tard.

(D) Non, car l'acte n'a pas été délivré effectivement

Raison d'être :
La bonne réponse estD. Cette question vous demande de prendre en compte l'exigence de livraison
des actes. Alf peut avoir eu l'intention de donner Blackacre à Becky, mais un acte doit être délivré
pour être effectif. La livraison exige qu'Alf renonce au contrôle de l'acte, ce qu'il n'a jamais fait. Il
aurait pu changer d'avis, sortir l'acte du tiroir et le détruire. L'affaireRosengrant c. Rosengrant traite
de la délivrance des actes et de la nécessité de renoncer au contrôle. La réponseA est incorrecte
car un acte ne nécessite pas de contrepartie pour être effectif. La réponseB est également
incorrecte car il n'est pas nécessaire d'enregistrer un acte pour qu'il prenne effet. Il est effectif au
moment de la remise, bien que l'enregistrement puisse certainement aider à établir que l'acte a été
remis et quand il l'a été. Comme indiqué ci-dessus, la réponseC est incorrecte car la donation n'a
pas été effectuée avant le décès d'Alf. L'acte n'ayant pas été effectivement délivré avant le décès
d'Alf, les Odd Fellows prendront Blackacre en vertu du testament.
Lecture recommandée :
Pages 394-397 de Dukeminier, Property, Concise Edition 2E
4

Amy habite sur le côté sud de la Route 1, en face de Jan et Mike Smith, un couple
marié qui habite sur le côté nord. Un jour qu'elle prenait un café avec les Smith sur leur
terrasse arrière, Mike a mentionné qu'il envisageait de vendre Blackacre, un terrain
d'environ 20 acres adjacent à celui d'Amy sur le côté sud de la Route 1, qu'il avait hérité
de son oncle. Amy a toujours voulu développer son exploitation laitière et a annoncé
aux Smith qu'elle souhaitait acheter le terrain. Amy déclare qu'ils se sont mis d'accord
sur un prix de 4000 dollars l'acre, le prix exact dépendant de l'arpentage du terrain pour
déterminer la superficie de la parcelle. Amy a écrit sur un bout de papier : "Amy
Johnson, acheteur, et Jan et Mike Smith, vendeurs/propriétaires. Propriété : 20 acres,
plus ou moins, sur le côté sud de la route 1, comté de Pawnee. Prix : 4000 $/acre.
Enquête à la charge d'Amy". Amy et Mike ont tous deux signé ce document. Jan n'a pas
signé. Amy déclare que Jan a dit : "C'est le département de Mike. Je n'ai jamais rien eu
à voir avec ces terres. Tout ce qu'il veut, c'est bien". Par la suite, Amy a engagé un
géomètre et Jan a rencontré le géomètre pour lui montrer l'emplacement de la propriété
à arpenter. L'arpentage a coûté 1 000 dollars et a montré que le terrain avait une
superficie d'environ 21 acres. Amy a engagé un avocat pour rechercher le titre de
propriété de Blackacre, ce qui lui a coûté 500 dollars. La recherche de titres montre que
le terrain est détenu par Mike et Jan Smith, en tant que locataires à part entière. Elle a
également payé 800 dollars pour la réparation de certaines clôtures afin que le terrain
soit prêt à accueillir son bétail dès la signature de l'accord. Lors de la clôture, Mike a
annoncé que l'opération devait être annulée parce que Jan refusait de signer l'acte, se
plaignant qu'elle n'avait jamais voulu vendre et qu'elle pensait que le terrain vaudrait
plus cher dans un avenir proche. Lequel des éléments suivants est vrai ?

(A) Amy peut poursuivre Jan en exécution forcée, car le papier est conforme à la loi sur
les fraudes (Statute of Frauds).

(B) Amy ne peut pas poursuivre Jan en exécution forcée, car le papier ne contient pas
une description légale correcte de Blackacre.

(C) Amy ne peut pas poursuivre Jan en exécution forcée, car le morceau de papier ne
contient pas le prix exact de la propriété.

(D) Amy devrait être en mesure d'obliger Jan à respecter le contrat, malgré la loi
sur les fraudes, en vertu d'une théorie de préclusion ou d'exécution partielle.

Raison d'être :
La bonne réponse estD. Pour les contrats immobiliers, la loi sur les fraudes (Statute of Frauds) exige
qu'il y ait un mémorandum de l'accord signé par la partie à lier, indiquant le prix, le bien et les
parties. Le document en question contient une description adéquate de la propriété ; il ne s'agit pas
nécessairement d'une description légale, mais simplement d'une description suffisante pour
permettre à un tribunal de déterminer la propriété visée. En l'espèce, la description du bien est
suffisante. La réponseB est donc incorrecte. En outre, même si le document n'indique pas le prix
exact, il suffit qu'il y ait une méthode permettant de déterminer le prix. La réponseC est donc
incorrecte car la méthode de calcul du prix est suffisamment explicitée. Cependant, la loi sur les
fraudes n'est pas respectée ici parce que Jan n'a pas signé le contrat et qu'elle est la "partie à lier".
Comme la propriété est détenue en tant que propriété entière, la transaction ne peut se faire sans
elle. Par conséquent, la réponseA est également incorrecte. Cependant, Amy a dépensé de l'argent
en se fiant raisonnablement à l'accord de Jan. Les dépenses qu'elle a engagées pour l'arpentage,
l'établissement du titre de propriété et les réparations de la clôture devraient suffire à établir
l'existence d'une préclusion ou d'une exécution partielle. Jan savait qu'Amy dépensait l'argent, mais
il a laissé faire.
Lecture recommandée :
Pages 362-366 de Dukeminier, Property, Concise Edition 2E
5

Betty Buyer a acheté une maison à Sally Seller, en prenant le titre de propriété par un
acte de garantie générale. La juridiction n'a pas d'obligation légale de divulgation
concernant l'état des locaux. L'acheteur a inspecté le bien avant la clôture, mais n'a rien
trouvé à redire. Cependant, peu de temps après avoir emménagé, elle a commencé à
se sentir mal. Au bout de quelques mois, elle a découvert qu'il y avait de la moisissure
derrière les panneaux du salon. Le vendeur avait posé des panneaux peu de temps
avant de vendre le bien et admet avoir vu des moisissures à l'époque. L'acheteur a reçu
une estimation de 20 000 dollars pour résoudre le problème. L'acheteur souhaite à
présent récupérer ce montant à titre de dommages et intérêts auprès du vendeur.
Lequel des éléments suivants est vrai ?
(A) La règle générale étant celle du caveat emptor, l'acheteur ne peut obtenir de
dommages-intérêts.

(B) L'acheteur peut obtenir des dommages-intérêts, car le défaut n'était pas
décelable par une inspection raisonnable.

(C) L'acheteur peut obtenir des dommages-intérêts, car le bien doit être exempt de tout
défaut matériel.

(D) L'acheteur peut obtenir des dommages-intérêts sur la base d'une fausse déclaration
affirmative du vendeur.
Raison d'être :
La bonne réponse estB. Un vendeur doit divulguer les défauts matériels dont il a connaissance et
qui ne peuvent être découverts lors d'une inspection raisonnable. Il s'agit d'une exception à la règle
du "caveat emptor", ce qui rend la réponseA incorrecte. La moisissure est un défaut matériel car elle
a un impact sur la valeur du bien. Il n'était pas possible de le découvrir parce qu'une inspection
raisonnable n'aurait pas consisté à déchirer des panneaux. C'était connu du vendeur car elle
l'admet. La réponseB est donc correcte. La réponseC est trop large ; il n'est certainement pas
nécessaire que le bien soit exempt de défauts matériels. Il peut y avoir des défauts, à condition qu'ils
soient patents ou divulgués par le vendeur, afin que les acheteurs puissent les prendre en compte
dans le prix de l'offre. La réponseD est erronée car nous n'avons aucune preuve que le vendeur ait
fait des déclarations concernant les moisissures ou la qualité générale de la maison.
Lecture recommandée :
Pages 371-378 de Dukeminier, Property, Concise Edition 2E
6

L'acheteur a conclu un contrat d'achat d'un ranch auprès du vendeur pour un montant
de 1 million de dollars. Le ranch se composait d'une vieille maison et d'une grange,
ainsi que d'une écurie pour les chevaux et de quelques pâturages. Le contrat précisait
dans sa clause 1 qu'il était "subordonné à l'obtention par l'acheteur d'un financement
acceptable". Le contrat ne contenait pas de clause d'inspection, mais précisait que
l'acheteur achetait le bien "en l'état". L'opération devait être conclue dans les 60 jours.
Au cours de la période de clôture, l'acheteur a découvert que le bardage de la grange
était pourri et qu'il faudrait probablement la démolir complètement dans un avenir
proche. La juridiction n'a pas d'obligation d'information pour ce défaut. L'acheteur a
demandé au vendeur de remédier à ce défaut ou de réduire le prix de 100 000 dollars.
Le vendeur a refusé. L'acheteur a alors notifié au vendeur qu'il résiliait le contrat en
raison du défaut de la grange et du fait qu'il n'avait pas obtenu de financement.
L'acheteuse a indiqué qu'elle avait fait plusieurs demandes auprès de plusieurs
banques. Compte tenu de sa cote de crédit, un seul a accepté de financer l'opération,
mais à hauteur de 80 % seulement du prix d'achat, à un taux supérieur au taux normal
pour un prêt hypothécaire de 30 ans. Le vendeur demande l'exécution forcée de
l'accord. Lequel des éléments suivants est le plus vraisemblablement vrai ?

(A) L'acheteur devrait être en mesure de résilier le contrat en raison du défaut de la


grange, mais pas du financement.
(B) L'acheteur devrait être en mesure de résilier le contrat sur la base du
financement, mais pas sur la base du défaut de la grange.

(C) L'acheteur devrait être en mesure de résilier le contrat sur la base du financement
ou du défaut de la grange.

(D) L'acheteur ne pourra pas annuler le contrat en raison d'un défaut de financement ou
d'un défaut de grange.
Raison d'être :
La bonne réponse estB. En ce qui concerne le défaut d'étable, l'obligation du vendeur se limite à
divulguer les défauts matériels dont il a connaissance et qui ne seraient pas révélés lors d'une
inspection raisonnable. Le défaut de la grange semble pouvoir être découvert facilement, il ne s'agit
donc pas d'un vice caché qui devait être divulgué. Le contrat immobilier ne contenait pas de clause
d'inspection ; au lieu de cela, l'acheteur acceptait les locaux "en l'état". Cela signifie qu'elle a
accepté de prendre en charge les vices cachés tels que le pourrissement du bardage de la grange.
Les réponsesA etC sont donc incorrectes. En ce qui concerne le financement, la clause de
contingence est rédigée de manière très large car elle ne précise pas les paramètres de
financement (taux d'intérêt, acompte, etc.) que l'acheteur doit accepter. Par conséquent, l'obligation
de l'acheteur se limite à faire "les meilleurs efforts" pour trouver un financement acceptable, ce
qu'elle a apparemment fait. Cette éventualité, rédigée en termes généraux, devrait lui permettre de
se retirer de l'accord et, par conséquent, la réponseD ne peut être correcte.
Lecture recommandée :
Pages 371-378 de Dukeminier, Property, Concise Edition 2E
7

Faust voulait acheter à Beowulf le lot 37 du Berkley Hills Addition. La propriété


comprend une maison à deux étages avec un garage détaché. L'adresse de la
propriété est 1839 Larson Street. Faust a fait une courte visite de la maison et l'a tout
de suite aimée. Faust s'apprêtait à partir pour l'Europe et les deux hommes n'avaient
pas de contrat en bonne et due forme à leur disposition : "Faust accepte d'acheter à
Beowulf le 1839 Larson Street pour 100 000 dollars. Beowulf accepte de fournir un titre
de propriété négociable. La clôture aura lieu dans deux mois". Les deux parties ont
signé et daté le document. À son retour d'Europe, quelques semaines plus tard, Faust a
examiné la maison de plus près et a découvert, dans une chambre, des taches et des
déformations dans le plâtre du plafond, ce qui indique que le toit fuit beaucoup. Elle
veut maintenant se retirer du contrat. En l'absence d'exigences légales, laquelle des
affirmations suivantes est vraie ?

(A) Le contrat de vente initial était insuffisant pour lier Faust à cet achat.

(B) Le toit qui fuit constitue une violation du titre de propriété négociable parce qu'il a un
impact matériel sur la valeur de la propriété, ce qui permet à Faust de résilier le contrat.

(C) La fuite du toit aurait dû être révélée à Faust, qui peut donc annuler.

(D) Faust n'a aucune raison valable de résilier le contrat.


Raison d'être :
La bonne réponse estD. La réponseA est incorrecte car le mémorandum écrit de l'accord répond
aux exigences de la loi sur les fraudes (Statute of Frauds) : Prix, Parties, Propriété. Le prix et les
parties sont clairement indiqués ; la seule question qui se pose est celle de la description du bien.
Bien que l'adresse de la rue ne soit pas une description légale correcte, elle est suffisante pour le
contrat car elle indique clairement quel bien est visé. La réponseB est incorrecte car un défaut
physique du bien n'a rien à voir avec un titre de propriété négociable. La réponseC est liée à la
doctrine de droit commun du caveat emptor. En common law, les acheteurs prenaient le bien en
l'état et supportaient le risque d'éventuels défauts. Les modifications modernes exigent la divulgation
des vices cachés, c'est-à-dire des problèmes qui ont un impact important sur la valeur du bien. Le
toit qui fuit est un défaut matériel, mais il n'est pas latent. Un vice caché est un vice connu du
vendeur mais qui ne peut être découvert par une inspection raisonnable. Ce défaut est patent, Faust
a pu constater que le plâtre était taché et gondolé. Par conséquent, Faust ne peut pas résilier le
contrat sur cette base. En règle générale, les contrats immobiliers contiennent une clause
d'inspection permettant à l'acheteur de disposer d'un certain délai pour procéder à une inspection
plus approfondie afin de déceler des défauts physiques. Ce contrat, cependant, ne comportait pas
une telle clause. En outre, de nombreux États disposent de lois exigeant la divulgation de certains
défauts matériels ; le problème vous demande d'ignorer cette possibilité. Par conséquent, Faust est
tenu par son marché.
Lecture recommandée :
Pages 362-379 de Dukeminier, Property, Concise Edition 2E
8

En 2000, Andrew a cédé Blackacre à sa fille Béatrice, sans contrepartie, par acte de
renonciation. En 2005, Beatrice a cédé Blackacre à Conner pour 500 000 dollars par un
acte de garantie générale. En 2015, Conner a voulu vendre Blackacre à Drake. Au
cours de la procédure de recherche de titres, Drake a découvert qu'Andrew était marié
à Winona au moment du transfert de propriété d'Andrew à Béatrice, bien que Blackacre
ait été titré au seul nom d'Andrew. Winona n'a pas participé à la transmission de
quelque manière que ce soit et est toujours en vie. Cette juridiction prévoit un droit de
douaire légal d'un tiers sur tout bien possédé par l'autre conjoint pendant le mariage.
Bien que Winona n'ait pas fait valoir ses droits et qu'Andrew soit toujours en vie, Drake
refuse de payer plus de 400 000 dollars pour la propriété. Conner peut prouver que la
juste valeur marchande de la propriété est de 600 000 dollars et que c'est le montant
que Drake a accepté de payer avant que la recherche de titres ne révèle l'intérêt
potentiel de Winona. La juridiction dispose d'un délai de prescription de cinq ans pour
les réclamations fondées sur la violation des garanties de propriété. Lequel des
énoncés suivants est vrai dans la plupart des juridictions ?

(A) Conner peut poursuivre Andrew, parce qu'il n'a pas obtenu la libération du droit de
douaire de Winona dans le transfert à sa fille Béatrice.

(B) Conner peut poursuivre Béatrice sur la base de la clause de saisine.

(C) Conner peut poursuivre Béatrice sur la base de l'engagement de garantie générale.
(D) Conner n'a pour l'instant aucune créance viable à l'encontre d'une
quelconque partie.

Raison d'être :
La bonne réponse estD. Cette question vous demande d'examiner l'effet des garanties dans les
actes et la nature du douaire. La réponseA est clairement erronée. Même si ce problème de titre est
en quelque sorte la "faute" de A, il n'a donné aucune garantie de titre. Un acte de renonciation dit en
substance : "Tout intérêt que je peux avoir est le vôtre". Elle ne fait aucune promesse quant à la
qualité du titre. Beatrice a toutefois remis à Conner un acte de garantie générale. Il s'agit d'un
ensemble d'engagements présents et futurs. La présente convention de saisine a été violée au
moment de la cession, car Béatrice n'avait pas de bon titre de propriété. Cependant, la violation a eu
lieu en 2005 et la prescription est acquise pour cette réclamation. La réponseB est donc incorrecte.
Les garanties futures, telles que l'engagement de garantie générale, ne sont violées que lorsque le
bénéficiaire de la garantie (Conner) est dérangé dans sa possession par quelqu'un qui revendique
un titre supérieur. Les faits indiquent que Winona n'a pas revendiqué son droit, et qu'elle ne le
pourrait pas non plus car il est inchoatif jusqu'au décès d'Andrew. La réponseC ne peut donc pas
être correcte. La bonne réponse est doncD, car à l'heure actuelle, Conner ne peut poursuivre
personne pour ce vice de titre.
Lecture recommandée :
Pages 388-394 de Dukeminier, Property, Concise Edition 2E
9

Joylon veut offrir Bleak House à son fils Junior. Il écrit sur un morceau de papier : "Acte
de propriété : Je, Joylon, exprime par la présente mon intention de donner à mon fils
Junior la propriété connue sous le nom de Bleak House, légalement décrite comme le
lot 1 de Copperfield Subdivision, Dickens County, Kansas." Il signe ce papier et le remet
à Junior. Quelques jours plus tard, Joylon meurt, laissant par testament tous ses biens
à sa fille Estelle. La juridiction n'autorisant pas les testaments olographes, Junior
prétend que le papier qu'on lui a remis était un acte. Dans un procès intenté par Estelle
pour revendiquer Bleak House, quelle est sa meilleure revendication concernant la
validité de la tentative de cession à Junior ?

(A) Le document n'a pas été notarié.

(B) Le document n'a pas été signé par Junior.

(C) Le document n'a pas utilisé les termes appropriés pour la transmission.

(D) Joylon aurait dû utiliser un acte officiel.


Raison d'être :
La bonne réponse estC. Le document n'a pas utilisé la terminologie du transfert actuel. Joylon a
exprimé son intention de céder la propriété à Junior à l'avenir. Un acte en bonne et due forme
indiquerait que Joylon "accorde, transfère et transmet" la propriété, ou plus simplement "transmet",
alors que cette formulation indique une intention de le faire, mais ne le fait pas réellement. Il est
possible que Junior puisse convaincre un tribunal de ne pas tenir compte de ce défaut, mais c'est
probablement le meilleur argument d'Estelle parmi ceux qui ont été donnés. En l'absence d'un
transfert en bonne et due forme de son vivant, Joylon est toujours propriétaire de Bleak House et le
titre de propriété est transmis à Estelle en vertu de son testament. Outre les mots de transmission,
les éléments d'un acte comprennent : une description du bien, l'identification du concédant et du
concessionnaire, et la signature du concédant. La validité d'un acte ne dépend pas de
l'authentification, bien que les lois exigent généralement l'authentification pour l'enregistrement de
l'acte. La réponseA est donc incorrecte. La réponseB est erronée car le preneur ne signe pas d'acte.
Il s'agit d'une action du concédant ; la signature du concessionnaire serait donc superflue. Enfin, la
réponseD est incorrecte car il n'est pas nécessaire que le concédant utilise un formulaire officiel ; un
acte peut être rédigé sur un bout de papier (même si cela n'est pas recommandé !).
Lecture recommandée :
Pages 388-390 de Dukeminier, Property, Concise Edition 2E
10

O voulait donner Blackacre à sa fille A à sa mort. Il a rédigé un acte de cession de


Blackacre à A et l'a remis à A en déclarant : "Je veux que tu aies ceci au cas où je
mourrais". A a rangé l'acte dans le placard. Plus tard, A est allée voir O et lui a dit
qu'elle aimerait que O ajoute son fils B à l'acte. O a accepté cette modification. Il
reprend l'acte et y inscrit le nom de B, de sorte qu'il se lit désormais "à A et B en tant
que copropriétaires". A a de nouveau mis l'acte dans le placard. O est ensuite décédé,
laissant un testament dans lequel il donnait tous ses biens à l'Église. A a sorti l'acte et
l'a montré à l'exécuteur testamentaire de O. Quel est l'état le plus probable du titre de
propriété de Blackacre, en common law ?

(A) L'Église est propriétaire de Blackacre, car l'acte de propriété n'a jamais été délivré
en bonne et due forme.

(B) A est propriétaire de Blackacre, car l'ajout ultérieur du nom de B a été


inefficace.

(C) A et B sont copropriétaires de Blackacre.

(D) L'Église est propriétaire de Blackacre, parce que le titre de propriété de A n'a pas
été enregistré et que l'ajout ultérieur de B a invalidé le titre de propriété.
Raison d'être :
La bonne réponse estB. O a passé un acte à A et le lui a donné. La remise de l'acte est remise en
cause parce que O semble l'assortir d'une condition orale. En common law, de telles conditions
orales ne sont pas valables parce qu'il y a trop de risques de fraude étant donné que la condition est
incompatible avec le contenu de l'acte. Certains tribunaux ont invalidé la livraison plutôt que la
condition, mais en common law, la livraison serait maintenue. La réponseA est donc incorrecte. O a
renoncé au contrôle de l'acte, contrairement à l'acte testamentaire dans l'affaire Rosengrant. Par
conséquent, O a correctement transmis Blackacre à A. Lorsque A a rendu l'acte à O afin qu'il puisse
ajouter B, O n'était plus propriétaire de Blackacre ; par conséquent, sa modification de l'acte n'a pas
eu d'effet. Bien qu'il n'ait eu aucun effet, il n'a pas invalidé l'acte comme le suggère la réponseD.
Une fois que l'acte a fait son travail de transmission du titre de O à A, ce qu'il advient de l'acte n'a
pas vraiment d'importance sur le plan juridique - l'acte est en effet terminé. Bien sûr, d'un point de
vue pratique, nous voulons conserver l'acte et l'enregistrer immédiatement pour le notifier ;
néanmoins, d'un point de vue juridique, il a fait son travail de transmission du titre de O à A. Afin
d'ajouter B en tant que co-locataire, A aurait dû transmettre, avec un nouvel acte, "à A et B en tant
que co-locataires" (ou dans certaines juridictions, transmettre à une personne de paille d'abord pour
respecter les unités de temps et de titre). Comme cela n'a pas été fait, A reste l'unique propriétaire
de Blackacre au décès de O, et la réponseC est incorrecte.
Lecture recommandée :
Pages 362-366 de Dukeminier, Property, Concise Edition 2E
11

Sayers conclut un contrat de vente de Blackacre, une parcelle de 80 acres située dans
une zone rurale, à Byers. Le contrat stipule que Sayers fournira "un titre de propriété
négociable, sous réserve de toutes les servitudes et charges enregistrées sur
Blackacre, légalement décrit comme W1/2 SE1/4 Section 16 dans le comté de Guthrie".
En examinant le titre de propriété, Byers découvre que Blackacre est grevé d'une
servitude de gazoduc. En outre, il apparaît que Sayers ne possède pas le titre de
propriété d'environ cinq acres de terrain (parcelle A) en bordure de la propriété décrite
dans le contrat. Sayers montre cependant à Byers qu'il est en possession exclusive et
continue de la parcelle A depuis plus de 20 ans. Étant donné que la période de
possession indésirable dans l'État n'est que de sept ans, Sayers affirme qu'il a établi un
bon titre de propriété sur la parcelle A. Byers refuse de conclure, alléguant l'absence de
titre négociable. Lequel des éléments suivants est vrai ?
C
(A) La servitude et la question de la parcelle A violent toutes deux les dispositions de ce
contrat relatives aux titres négociables.

(B) La servitude viole la disposition du contrat relative aux titres négociables, ce qui
n'est pas le cas de la parcelle A.

(C) La servitude n'enfreint pas la disposition du contrat relative aux titres


négociables ; la question de la parcelle A, elle, l'enfreint.

(D) Ni la servitude ni la question de la parcelle A n'enfreignent la disposition du contrat


relative aux titres négociables.
Raison d'être :
La bonne réponse estC. Le titre de propriété négociable exige que le vendeur établisse qu'il est le
propriétaire enregistré, en pleine propriété absolue, du bien décrit, sans charges. Dans le cas
présent, il existe deux violations possibles de cette norme : la servitude et l'absence
d'enregistrement du titre de propriété de la parcelle A. En vertu de la common law, ces deux
questions violeraient la norme relative au titre de propriété négociable. Cependant, dans ce cas,
Byers a choisi d'accepter un contrat dont les termes étaient légèrement différents, une promesse de
fournir "un titre de propriété commercialisable sous réserve de toutes les servitudes et de tous les
encombrements enregistrés". La servitude de pipeline étant enregistrée, elle n'enfreint pas les
dispositions relatives aux titres négociables de ce contrat. Les réponsesA etB sont donc incorrectes.
La question de la parcelle A, cependant, n'est pas couverte par cette exception contractuelle. Même
si Sayers peut, en fait, avoir un titre de propriété sur la parcelle A, elle n'a pas de "titre négociable"
parce qu'un titre négociable ne peut pas être soumis à une perspective non frivole de litige sur la
propriété. La réponseD est donc incorrecte. Sayers doit soit faire annuler le titre de propriété de la
parcelle A avant d'essayer de la vendre, soit obtenir d'un acheteur potentiel qu'il fasse une exception
spécifique à la disposition relative au titre de propriété négociable.
Lecture recommandée :
Pages 366-370 de Dukeminier, Property, Concise Edition 2E
12

Stephen Gates a acheté Blueacre en 2001 pour 10 millions de dollars. Il a payé un


million de dollars en espèces et a financé le reste par un prêt de la First Bank (FB) en
lui accordant une hypothèque qui a été rapidement enregistrée. En 2005, Gates a
contracté un autre prêt, cette fois auprès de la Second Bank (SB), qui l'a garanti par
une hypothèque sur Blueacre, également enregistrée sans délai. Peu de temps après,
M. Gates n'a pas remboursé son prêt. Il devait encore 7 millions de dollars à FB et 1
million de dollars à SB. Les deux banques ont saisi Blueacre et le tribunal a procédé à
la vente de la saisie. Le seul enchérisseur lors de la vente sur saisie a été Bill Jobs, qui
a acheté la propriété pour 6 millions de dollars. Lequel des éléments suivants est vrai ?

(A) Si cet État a adopté une législation anti-déficit, FB et SB devraient pouvoir


poursuivre Gates pour le reste de ce qu'il doit.

(B) Dans la plupart des États, la vente peut être annulée si le tribunal n'a pas pris de
mesures suffisantes pour obtenir un prix équitable pour le bien.

(C) Même après la vente à Jobs, certains États autorisent Gates à racheter la
propriété en payant le prix de vente de 6 millions de dollars.

(D) SB devrait avoir droit à une part proportionnelle du produit de 6 millions de dollars.
Raison d'être :
La bonne réponse estC. Cette question teste vos connaissances sur divers aspects de la procédure
de saisie, en particulier le droit de rachat et la priorité de paiement entre les créanciers
hypothécaires. Dans ce scénario, il y a une première hypothèque et une deuxième hypothèque qui a
une priorité inférieure. La réponseA est incorrecte car la législation anti-déficit limite le créancier
hypothécaire au bien garantissant la dette ; par conséquent, dans ces juridictions, les prêteurs ne
pouvaient pas poursuivre Gates pour les montants restants dus. En ce qui concerne la réponseB,
dans certaines juridictions, les créanciers hypothécaires qui procèdent à des saisies non judiciaires
sont tenus de prendre des mesures pour garantir un prix équitable. Cependant, il s'agit d'une saisie
judiciaire, effectuée par le tribunal lui-même, et la seule question serait donc de savoir si le tribunal a
suivi les procédures prévues par la loi. Rien n'indique dans le problème que les exigences légales
n'ont pas été respectées. La réponseC fait référence au droit légal de rachat prévu dans certains
États, qui permet au débiteur de racheter le bien dans un certain délai après la vente par saisie. La
réponseC énonce correctement la loi. La réponseD est incorrecte car la priorité des privilèges
détermine qui est payé en premier. Dans cette situation, la première banque est prioritaire et serait
entièrement remboursée avant que la deuxième banque ne reçoive quoi que ce soit. Il n'y a pas de
partage proportionnel du produit de la saisie.

Chapitre 8

En 2010, Whistler a emprunté 50 000 dollars à sa mère, Ma, qui vivait à proximité. Il a
signé un billet et une hypothèque sur Blackacre, une propriété locative qu'il possédait,
pour garantir le prêt. Ma n'a cependant pas enregistré l'hypothèque à ce moment-là. Le
1er mars 2016, Whistler a conclu un contrat de vente de Blackacre à Bono. Bono a
effectué une recherche de titres qui, bien entendu, n'a pas révélé l'existence de
l'hypothèque à Ma et il n'a pas été informé de son intérêt. La vente a été conclue le 1er
avril 2016. Bono a payé la valeur totale du bien (200 000 dollars) en échange d'un acte
de garantie générale. Whistler a dit à Ma qu'il la paierait sur le produit de la vente, mais
il ne l'a pas fait. Au lieu de cela, il a pris l'argent et est parti pour l'Europe. Le 15 avril
2016, Ma a enregistré son hypothèque. Le 30 avril 2016, Bono a enregistré l'acte de
garantie générale. Ma cherche maintenant à saisir Blackacre pour satisfaire son
hypothèque. Lequel des éléments suivants est vrai ?

(A) Si l'État dispose d'une loi sur l'enregistrement des avis de race, Ma ne pourra pas
saisir le bien parce que son intérêt n'a pas été enregistré lorsque Bono a acheté
Blackacre.

(B) Si l'État dispose d'une loi sur l'enregistrement de type notification, Ma ne


pourra pas saisir Blackacre, même si elle a enregistré en premier.

(C) À moins que l'État ne dispose d'une loi sur l'enregistrement des races, Ma ne pourra
pas saisir Blackacre.

(D) Quel que soit le type d'acte d'enregistrement, le droit de Ma ne survivrait pas au
transfert de propriété à Bono ; son seul recours est de poursuivre Whistler sur le billet.
Raison d'être :
La bonne réponse estB. Dans un problème d'acte d'enregistrement, dessinez un petit diagramme
montrant quand les transports et les enregistrements ont eu lieu. Dans ce cas, Whistler a transmis à
Ma, puis Whistler a transmis à Bono, puis Ma a enregistré, puis Bono a enregistré. Étant donné que
le transfert à Ma a été effectué en premier lieu, Ma pourra saisir le bien à moins que Bono ne
satisfasse aux exigences de la loi sur l'enregistrement de la juridiction. S'il s'agit d'un acte de
notification, Bono gagne parce qu'il a pris sans notification de la transaction précédente. S'il s'agit
d'un acte de notification de la race, Bono perd car même s'il a pris sans notification, il doit également
enregistrer en premier, ce qu'il n'a pas fait. De même, en vertu d'une loi purement raciale, Bono perd
parce que Ma a enregistré en premier. Par conséquent, la réponseA est incorrecte car Ma serait en
mesure de saisir dans un État où la race est notifiée. Les réponsesC etD sont également incorrectes
pour la même raison. La réponseB est le seul énoncé correct de la loi.
Lecture recommandée :
Pages 437-439, 446-451 de Dukeminier, Property, Concise Edition 2E
2

Joy possédait une maison appelée Greenacre. Lorsque sa mère, Dawn, est devenue
fragile, Joy l'a invitée à vivre avec elle à Greenacre. Elle a rédigé un acte accordant à
sa mère un droit viager sur Greenacre et le lui a remis en déclarant : "Si quelque chose
devait m'arriver, je veux que tu puisses vivre ici pour toujours". Dawn a pris l'acte et l'a
rangé dans le tiroir de son bureau. Peu après, Joy est tombée amoureuse de Rock et l'a
épousé. Elle a cédé Greenacre "de Joy à Joy et Rock en tant que copropriétaires avec
droit de survie". Elle a déclaré à Rock : "Bien sûr, cela est soumis au droit de ma mère
de vivre sa vie ici ; je lui ai donné un intérêt viager". Rock a répondu : "Bien sûr, pas de
problème" et a immédiatement enregistré cet acte. Tragiquement, Joy a été tuée
quelques mois plus tard dans une avalanche alors qu'elle faisait du ski. Rock n'a pas
l'intention de vivre à Greenacre avec sa belle-mère. Il décide de vendre Greenacre à
son ami Hardplace. Il a dit à Hardplace : "Écoutez, Dawn a apparemment un droit
viager, mais comme il n'est pas enregistré, vous devriez pouvoir vous débarrasser d'elle
immédiatement." Il a cédé à Hardplace, par acte de renonciation, la totalité de la valeur
marchande de Greenacre, et Hardplace a immédiatement enregistré cet acte. Lequel
des éléments suivants est vrai dans le cadre d'un procès intenté par Hardplace en vue
d'expulser Dawn de Greenacre ?

(A) Hardplace ne gagnera que dans le cadre d'une course pure enregistrant la
compétence de l'acte.

(B) Hardplace ne perdra que dans le cadre d'une loi sur l'enregistrement des avis.

(C) Hardplace perdra dans toutes les juridictions.

(D) Hardplace gagnera dans toutes les juridictions.


Raison d'être :
La bonne réponse estA. Dawn semble être entre le marteau et l'enclume, mais l'est-elle vraiment ?
Joy a correctement transmis un intérêt vital à Dawn. Elle a ensuite transmis un demi-intérêt avec
droit de survie à Rock, mais comme le transfert à Dawn a eu lieu en premier, en common law,
l'intérêt de Rock est grevé par le domaine viager. Au décès de Dawn, Rock est propriétaire du fief
simple, mais il est toujours grevé par le domaine viager précédemment cédé. Lorsqu'il la transmet à
Hardplace, elle reste grevée, en common law, par le domaine viager. Par conséquent, la seule façon
pour Hardplace de se libérer des intérêts de Dawn est de se conformer aux dispositions de la loi sur
l'enregistrement. Dans une juridiction de notification ou de race-notice, un acquéreur ultérieur à titre
onéreux doit prendre sans notification ; il est clair que Hardplace avait une notification en l'espèce
puisqu'il a été informé de l'intérêt de Dawn. La réponseB est donc incorrecte, car H perdra
également dans une juridiction soumise à un avis de race. En outre, Dawn occupe la propriété, ce
qui soulève la question de l'avis d'enquête. Toutefois, dans une juridiction dotée d'un statut
purement racial, nous ne nous soucions pas de ce que savait Hardplace. Tant qu'il enregistre son
acte en premier, ce qu'il a fait, et ce que Rock a fait, il devrait gagner. Vous pourriez penser que
Dawn perdrait dans n'importe quelle juridiction parce qu'elle n'a pas payé la valeur et n'a pas
enregistré, mais rappelez-vous que ni l'enregistrement ni la contrepartie ne sont nécessaires pour un
transfert valide ; la réponseD est donc incorrecte. On pourrait penser que Dawn gagnerait dans
n'importe quelle juridiction en raison de la connaissance de Hardplace et Rock, mais encore une
fois, la loi sur la course pure ne tient pas compte de cela, se concentrant sur les avantages de la
certitude de s'appuyer uniquement sur la date des documents enregistrés, et donc, la réponseC est
incorrecte.
Lecture recommandée :
Pages 437-439, 446-451 de Dukeminier, Property, Concise Edition 2E
3

Oscar fait don de Blackacre à Alice, qui n'enregistre pas l'acte. Alice vend alors
Blackacre pour sa valeur à Bailey, qui enregistre immédiatement cet acte. Bailey n'a
pas pris possession de Blackacre parce qu'elle habitait loin ; elle avait simplement
l'intention de le détenir à des fins d'investissement. Elle a laissé Oscar continuer à y
vivre, sans payer de loyer, pendant qu'elle décidait de ce qu'elle allait faire de la
propriété. Quelques années plus tard, Oscar a eu besoin d'argent et a obtenu un prêt
de la First Bank en lui accordant une hypothèque sur Blackacre. La First Bank n'était
pas au courant des transferts antérieurs. Peu de temps après, Oscar n'a pas remboursé
le prêt, ayant dépensé tout l'argent qu'il avait emprunté. La First Bank cherche
maintenant à saisir Blackacre. Bailey s'y oppose au motif qu'Oscar n'était plus
propriétaire de Blackacre lorsqu'il a donné l'hypothèque à la First Bank. Dans une
juridiction de notification qui utilise un indice concédant-concessionnaire, quelle est la
réponse à la question suivante ?

(A) La plupart des tribunaux donneraient raison à la First Bank, car elle n'a pas
été informée de la cession à Bailey.

(B) La plupart des tribunaux donneraient raison à Bailey, car c'est elle qui a enregistré
le premier et, par conséquent, la First Bank a bénéficié d'une notification implicite.

(C) La plupart des tribunaux donneraient raison à la First Bank, car Alice n'était pas un
acquéreur à titre onéreux.

(D) La plupart des tribunaux donneraient raison à la First Bank, car Alice ne pouvait pas
transmettre un titre valable à Bailey tant qu'elle n'avait pas enregistré l'acte d'Oscar.
Raison d'être :
La bonne réponse estA. Pour répondre aux problèmes liés à l'acte d'enregistrement, vous devez
d'abord vous demander qui gagne en l'absence d'acte d'enregistrement. (Il est également utile de
dessiner un diagramme avec des flèches indiquant qui a fait quoi et quand). Dans ce cas, Oscar a
transmis à Alice, qui a ensuite transmis à Bailey. Oscar a ensuite accordé une hypothèque à la First
Bank. En l'absence d'un acte d'enregistrement, Bailey gagne parce qu'elle était la première dans le
temps. Au moment où Oscar a hypothéqué Blackacre, il n'en était plus propriétaire. Par conséquent,
la seule façon pour la First Bank d'obtenir gain de cause est de se qualifier en tant qu'acheteur de
bonne foi (BFP) en vertu de la loi sur l'enregistrement de la juridiction. Le problème est qu'il s'agit
d'une juridiction de notification, ce qui signifie que la First Bank doit être un acquéreur ultérieur à titre
onéreux qui a pris possession sans notification de la transaction précédente. Le terme "acheteur"
englobe généralement les créanciers hypothécaires tels que la First Bank, de sorte que la seule
véritable question est de savoir si la banque était informée. Vérifier les trois types de notification :
réelle, implicite (enregistrement) et enquête. Le problème est que la First Bank n'a pas été
réellement avertie. En outre, il n'y a aucune raison de penser qu'une quelconque circonstance
mettrait la First Bank à l'enquête. Le problème est qu'Oscar est toujours en possession du bien. (Par
conséquent, la seule question qui se pose est de savoir si l'enregistrement de Bailey a donné à la
banque une notification constructive (enregistrement). Son acte a été enregistré lorsque la First
Bank a pris l'hypothèque, mais pas de manière à ce que la banque puisse le retrouver. Dans un
index des cédants, la First Bank chercherait le nom d'Oscar et ne trouverait rien, car l'acte d'Oscar à
Alice n'a pas été enregistré. L'acte Alice-Bailey a été enregistré, mais uniquement sous les noms
d'Alice et de Bailey ; il n'y a aucune raison pour que la First Bank recherche leurs noms. Par
conséquent, la plupart des tribunaux estimeraient que la First Bank est un BFP et qu'elle peut donc
saisir Blackacre à moins que le prêt hypothécaire ne soit remboursé. La réponseB est incorrecte car
même si l'acte de A à B a été enregistré, il était en dehors de la chaîne de titres et, par conséquent,
n'a pas donné lieu à une notification implicite. La réponseC est incorrecte car il importe peu qu'Alice
(ou Bailey, d'ailleurs) ait payé la valeur. Dans une question relative à l'acte d'enregistrement, seul
l'acquéreur ultérieur doit payer la valeur. La réponseD est incorrecte car l'enregistrement n'est pas
nécessaire pour transmettre un bon titre de propriété. L'enregistrement ne fait que donner un avis ; il
n'affecte pas la validité de la transaction.
Lecture recommandée :
Pages 437-439, 446-451 de Dukeminier, Property, Concise Edition 2E
4

Oscar possédait Blackacre qui était adjacent à Whiteacre, propriété de l'Église. Les
membres de l'église ont utilisé une partie de Blackacre, appelée parcelle A, comme
parking pendant de nombreuses années. Oscar a décidé de donner la parcelle A à
l'Église, et lui a donc donné un acte de donation pour cette partie de Blackacre. L'Église
n'a cependant pas enregistré cet acte. Oscar a vendu Blackacre à Able et lui a parlé de
l'acte précédent concernant la parcelle A. Oscar l'a cédée à Able par acte de
renonciation. Plusieurs mois plus tard, Able vend Blackacre à Baker, pour la valeur
totale de Blackacre, y compris la parcelle A. Able dit à Baker qu'il pourrait y avoir des
problèmes en ce qui concerne le titre de la parcelle A, mentionnant que " Oscar pense
qu'il l'a donné à l'Église d'une manière ou d'une autre ". Cependant, Baker a fait des
recherches sur le titre et n'a rien trouvé d'inquiétant. Able a ensuite transmis Blackacre
à Baker par un acte de garantie générale, que Baker a rapidement enregistré. Baker a
ensuite tenté d'empêcher l'Église d'utiliser la parcelle A, ce à quoi l'Église a répondu en
lui montrant l'acte de donation d'Oscar. Baker poursuit maintenant Able et l'Église ; il
veut soit que la revendication de l'Église sur la parcelle A soit annulée, soit qu'Able lui
verse des dommages-intérêts. Dans une juridiction de notification, lequel des points
suivants est vrai ?

(A) Baker n'a aucun droit contre Delta en vertu de l'acte de garantie générale, parce
que Delta lui a parlé du problème potentiel de la parcelle A.

(B) L'Église n'a pas un bon titre de propriété sur la parcelle A, parce qu'elle ne l'a pas
enregistré et qu'elle n'est pas un acquéreur ultérieur à titre onéreux.

(C) Baker perdra contre l'Église, parce qu'il avait au moins un avis d'enquête sur
la demande de l'Église.

(D) Baker a une créance sur Oscar en vertu des garanties de l'acte, car Oscar a créé le
problème de titre.
Raison d'être :
La réponse correcte estC. La première question est de savoir si l'Église a un bon titre de propriété
sur la parcelle A malgré son absence d'enregistrement. Dans une juridiction de notification, Baker
doit être un acquéreur ultérieur à titre onéreux qui a pris la décision sans notification. Baker a acheté
pour la valeur, et cette partie du test BFP a donc été remplie. Toutefois, il a reçu un avis d'enquête
concernant l'intérêt de l'Église. Même s'il a recherché le titre de propriété et n'a pas trouvé l'acte, il
aurait dû le demander à l'Église sur la base du commentaire d'Able, d'autant plus que l'Église était
en possession de l'acte. Par conséquent, la réponseB est incorrecte et la réponseC est correcte. En
ce qui concerne les garanties de l'acte, Baker devrait pouvoir poursuivre Able en vertu du pacte de
saisine ou du pacte de jouissance paisible. À moins qu'Able n'ait spécifiquement exclu la parcelle A
des garanties de l'acte, il garantit un bon titre de propriété, indépendamment de ce qu'il a dit à
Baker. La réponseA est donc incorrecte. La réponseD est également incorrecte car Oscar n'a pas
garanti le titre de propriété ; il a donné un acte de renonciation, qui ne contient aucune garantie.

Chapitre 10
1

Abdul est propriétaire de Blackacre, un terrain de 40 acres situé dans une zone
agricole, adjacent à Whiteacre, qui appartient à son bon ami Nguyen. Pendant de
nombreuses années, Abdul a permis à Nguyen de traverser Blackacre avec son bétail
et son matériel agricole pour atteindre un autre champ appartenant à Nguyen de l'autre
côté de Blackacre, appelé Greenacre. Abdul s'apprête à vendre Blackacre à Betsy et
veut s'assurer que Nguyen pourra continuer à utiliser le droit de passage. Lequel des
moyens suivants serait lapire façon pour Abdul d'y parvenir ?

(A) Abdul pourrait rédiger et exécuter une servitude expresse en faveur de Nguyen et
l'enregistrer, puis céder la propriété à Betsy "sous réserve de toutes les charges
enregistrées".

(B) Abdul pourrait céder Blackacre à Betsy "sous réserve d'une servitude en
faveur de Nguyen pour l'entrée et la sortie".

(C) Abdul pourrait céder Blackacre à Betsy "en réservant au concédant une servitude
d'entrée et de sortie entre Whiteacre et Greenacre". Abdul pourrait alors céder cette
servitude à Nguyen.

(D) Abdul pourrait dire à Betsy qu'il ne lui vendra que si elle parvient à un accord avec
Nguyen concernant l'accès et en faire une condition de la vente. À la clôture, Abdul
cède le titre de propriété en pleine propriété à Betsy, qui cède ensuite une servitude à
Nguyen.
Raison d'être :
La bonne réponse estB. La réponseA est la manière la plus propre d'effectuer cette transaction. Une
fois qu'une servitude est créée, elle grève Blackacre et Abdul devrait simplement indiquer dans
l'acte qu'il transmet un titre en pleine propriété à l'exception de cette servitude ou d'autres servitudes
enregistrées. La réponseB est la plus problématique car de nombreuses juridictions adhèrent à
l'ancienne règle de common law interdisant la création d'une servitude dans un "étranger à l'acte"
(c'est-à-dire quelqu'un d'autre que le concédant et le concessionnaire). La règle est assez logique :
pour créer une servitude expresse, il faut délivrer l'acte de concession au preneur. Ici, l'acte est
remis à Betsy, et non à Nguyen. Certaines juridictions ont abandonné cette règle parce qu'elle
contrecarre l'intention claire du concédant, mais ce n'est pas un très bon moyen d'y parvenir, même
dans les juridictions où elle pourrait être autorisée, parce qu'elle brouille les titres de propriété. La
procédure décrite dans la réponseC fonctionnerait. Le concédant peut certainement se réserver une
servitude et la transmettre ensuite. Ce n'est cependant pas la meilleure façon de procéder, car cela
créerait une confusion quant à la propriété qui constitue la parcelle dominante. La réponseD serait
également valable et pourrait même être préférable si Betsy voulait négocier avec Nguyen les
conditions de la servitude (étendue, emplacement, etc.).
Lecture recommandée :
Pages 485-492 de Dukeminier, Property, Concise Edition 2E
2

Cora possède des propriétés adjacentes, Blackacre et Whiteacre. En 2010, elle a


transmis Blackacre à son fils Devin. Elle a inclus dans l'acte une servitude sur
Whiteacre afin que Devin puisse facilement rejoindre la route principale. En 2012, Devin
a vendu Blackacre à Edie. Son acte de cession de Blackacre à Edie ne mentionne pas
la servitude sur Whiteacre et elle n'a pas tenté de l'utiliser. En 2014, Edie a rétrocédé
Blackacre à Cora. Là encore, il n'y avait aucune mention de la servitude dans l'acte. En
2016, Cora a cédé Blackacre à Fagin, mais n'a pas inclus la servitude dans l'acte. Fagin
veut utiliser la servitude qu'il a découverte lors de la recherche de titres. Tous les actes
susmentionnés ont été correctement enregistrés. Cora prétend que la servitude n'est
plus valable. Qui gagne ?

(A) Cora, parce que la servitude a pris fin en 2012, lorsque Devin l'a cédée à Edie.

(B) Cora, parce que la servitude a pris fin en 2014, suite à une fusion

(C) Cora, parce que la servitude a pris fin en raison de l'abandon.

(D) Fagin, parce que la servitude est rattachée à Blackacre


Raison d'être :
La bonne réponse estB. Selon la doctrine de la fusion, une servitude s'éteint lorsque la parcelle
dominante et la parcelle servante appartiennent au même propriétaire, car il n'est pas possible
d'avoir une servitude sur sa propre terre. Par conséquent, lorsque Edie a rétrocédé son bien à Cora
en 2014, la servitude a pris fin. Lorsque Cora a cédé son bien à Fagin, elle aurait dû recréer la
servitude, ce qu'elle n'a pas fait. La réponseD est donc incorrecte. La réponseA est incorrecte, car
cette servitude est rattachée à Blackacre et inhérente à son titre ; elle n'est pas personnelle à Devin.
Par conséquent, elle est restée valable lorsque Devin l'a transmise à Edie, même s'il ne l'a pas
mentionnée dans l'acte. La réponseC est incorrecte car, même si Edie n'a pas utilisé la servitude,
rien n'indique qu'elle avait l'intention de renoncer à ses droits sur celle-ci. La simple non-utilisation
ne constitue pas un abandon.
Lecture recommandée :
Page 532 de Dukeminier, Property, Concise Edition 2E
3

En 1990, Albert était propriétaire de Whiteacre et Bertil du terrain adjacent, Blackacre.


Albert accorde à Bertil une servitude sur Whiteacre " dans le but d'exploiter une
gravière sur Blackacre ". La servitude a été enregistrée en bonne et due forme. De
2000 à 2015, Bertil a fait un usage sporadique de la servitude, la gravière devenant
progressivement moins rentable. L'herbe a poussé sur la piste de terre et était à peine
visible en 2010. En 2010, Albert a cédé Whiteacre à Candace, mais n'a pas mentionné
la servitude, que ce soit oralement ou dans l'acte. En 2017, Bertil n'avait pas utilisé la
servitude depuis de nombreuses années. Bertil a déclaré à plusieurs personnes qu'une
fois que le prix du gravier aura augmenté, il pourrait recommencer à utiliser la propriété
pour l'exploitation minière. En 2017, il a rempli la fosse d'eau et a commencé à la louer
à des groupes pour des activités de camping et de sports nautiques. L'utilisation de la
servitude a donc augmenté de manière significative. Candace a intenté un procès à
Bertil pour l'empêcher d'utiliser la servitude. Lequel des éléments suivants est le plus
susceptible d'aboutir dans cette affaire ?

(A) Le tribunal estimera que la servitude a pris fin lorsque Albert a vendu Whiteacre à
Candace.

(B) Le tribunal considérera que la servitude a été supprimée par abandon.

(C) Le tribunal interdira à Bertil d'utiliser la servitude pour un nouvel usage.

(D) Le tribunal estimera que Bertil dispose d'une servitude de passage expresse qui
peut être utilisée à cette fin, ce qui constitue une "évolution naturelle" de l'utilisation
précédente.
Raison d'être :
La bonne réponse estC. La servitude existe toujours, mais sa portée est limitée. La réponseA est
incorrecte car la servitude est une charge sur Blackacre et reste en vigueur tant que l'acquéreur
ultérieur en est informé. Il a été enregistré ici, et est donc resté une charge lorsque Candace en a
pris possession. La question de l'abandon est plus délicate ; bien qu'il s'agisse d'une longue période
d'inutilisation, le tribunal doit disposer d'indications, explicites ou implicites, selon lesquelles Bertil
avait l'intention de renoncer à ses droits. Un tribunal pourrait estimer que l'utilisation de la propriété à
des fins récréatives pourrait indiquer que Bertil avait l'intention d'abandonner l'exploitation du
gravier, mais il existe des preuves qu'il s'agissait d'une cessation temporaire. (Il a dit aux gens qu'il
pourrait reprendre l'exploitation lorsque le prix du gravier augmenterait). Ainsi, bien que la servitude
existe encore très probablement, l'utilisation actuelle dépasse le champ d'application de la servitude.
La formulation limite clairement l'utilisation "pour l'exploitation d'une gravière". Bien qu'une certaine
"évolution naturelle" d'une servitude soit autorisée, l'utilisation à des fins récréatives est
complètement différente de l'exploitation minière et présente des charges différentes sur le fonds
servant. Par conséquent, un tribunal interdirait probablement la nouvelle utilisation de Bertil, mais
considérerait que la servitude d'exploitation minière existe toujours.
Lecture recommandée :
Pages 515-523 de Dukeminier, Property, Concise Edition 2E
4

En 2012, Kevin a acheté le lot 1 du lotissement Greenacres dans la ville d'Anytown.


Lorsque la subdivision a été tracée à l'origine en 1927, le promoteur a placé une clause
restrictive sur tous les lots, les limitant à un usage résidentiel uniquement. En 1955,
Anytown a zoné la propriété R-1, qui n'autorise que l'usage résidentiel unifamilial. La
rue sur laquelle se trouve la propriété de Kevin, qui était autrefois une route à deux
voies, est aujourd'hui devenue une grande artère, et plusieurs entreprises ainsi qu'une
grande église se trouvent maintenant de l'autre côté de la rue du lotissement. Une
habitante d'une rue intérieure utilise sa maison pour une petite crèche, et une autre
habitante est comptable et reçoit ses clients chez elle. Kevin aimerait utiliser sa maison
sur le lot 1 pour y installer un cabinet d'avocats qui, selon lui, aura peu de circulation et
ne sera pas bruyant. Il peut prouver que la maison vaut 200 000 dollars en tant que
résidence, mais environ 300 000 dollars si une telle utilisation commerciale est
autorisée. Ses voisins de la subdivision de Greenacres s'y opposent cependant et
déclarent qu'ils ont l'intention de faire respecter la convention contre l'utilisation qu'il se
propose de faire. Si vous représentiez Kevin, quel serait votre meilleur argument
concernant l'applicabilité de la convention ?

(A) La convention restreignant la propriété de Kevin doit être considérée comme


inapplicable en raison d'un "changement de circonstances".

(B) La convention ne peut être opposée à Kevin, car il n'y a pas de lien de connexité
horizontale.

(C) La convention doit être considérée comme abandonnée.

(D) La convention ne peut être opposée à Kevin, car les voisins ne peuvent plus
prouver l'existence d'un plan de développement commun.
Raison d'être :
La bonne réponse estC. La réponseC est le meilleur argument car l'existence de la crèche et du
bureau du comptable peut être la preuve que la convention a été levée. La renonciation peut être
invoquée avec succès si les violations antérieures d'une convention sont similaires à la violation
proposée en ce qui concerne les préjudices qu'elles causent. En l'espèce, l'utilisation proposée par
Kevin n'entraînerait qu'une faible augmentation du trafic, probablement pas plus importante que
celle causée par la crèche et le bureau du comptable, et ne causerait aucun autre préjudice aux
autres propriétaires du lotissement. La réponseA exige la preuve d'un "changement de
circonstances", ce qui n'est pas une mince affaire. Les tribunaux exigent généralement que les
changements aient contrecarré l'objectif de la convention, ce qui ne semble pas être le cas en
l'espèce. En outre, les changements doivent avoir lieu à l'intérieur du lotissement plutôt qu'à
l'extérieur, ce qui s'est produit dans le cas présent. A est donc un argument possible, mais pas très
convaincant. La réponseB ne tient pas la route : il y a eu un lien de connexité horizontal (entre le
donneur d'ordre et le preneur d'ordre). La réponseD est également incorrecte ; l'argument du régime
commun est utilisé pour impliquer une convention sur des lots qui ne font pas l'objet d'une
convention explicite, comme c'est le cas du lot de Kevin.
Lecture recommandée :
Pages 545-551 de Dukeminier, Property, Concise Edition 2E
5

Mia possédait Blackacre, qui était bordé à l'est par Whiteacre, propriété de Nelson. Les
deux propriétés sont bordées par River Road au sud et par la Big River au nord. Un
jour, Nelson dit à Mia qu'il envisage de construire une nouvelle maison plus loin sur
Whiteacre, sur une crête avec une bonne vue sur la rivière en contrebas.
"Malheureusement, il n'y a pas de bon moyen d'y retourner sur ma propriété à cause
d'un grand ravin et d'une zone humide. Ça te dérange si je passe par Blackacre pour y
retourner ?" "Bien sûr, je peux te laisser faire, Nelson", a répondu Mia. L'entreprise de
construction engagée par Nelson a construit un chemin de terre à l'extrême est de
Blackacre et l'a utilisé pour se rendre sur le chantier où elle a construit une belle
maison. Nelson s'y installe et continue d'utiliser le chemin de terre au-dessus de
Blackacre pendant les huit années suivantes. Nelson mourut ensuite et sa fille Olivia
hérita de Whiteacre. Elle a également utilisé la route pendant un an. Mia a ensuite
vendu Blackacre à Portia. Portia s'est vite lassée de voir un étranger circuler sur sa
propriété et a bloqué l'accès d'Olivia. Portia prétend que Nelson disposait tout au plus
d'une licence personnelle et révocable. Olivia souhaite intenter une action pour établir
son droit à la servitude. Le délai de prescription est de sept ans dans cette juridiction.
Laquelle des théories suivantes serait la plus susceptible d'aider Olivia dans ce cas ?

(A) Servitude par prescription

(B) Servitude par préclusion

(C) Servitude implicite par nécessité

(D) Servitude implicite en raison d'un usage antérieur


Raison d'être :
La bonne réponse estB. Une servitude par préclusion correspond aux faits : elle exige que le
propriétaire de la parcelle dominante (Nelson) se fie de manière raisonnable et prévisible à une
promesse du propriétaire de la parcelle servante (Mia). Mia a promis à Nelson qu'il pourrait utiliser la
route, et il s'est appuyé sur cette promesse pour construire une maison coûteuse. Par conséquent,
la licence peut s'être transformée en une servitude irrévocable, de sorte que Portia est désormais
liée par elle. Bien que le problème ne le demande pas, Portia serait liée si elle prenait avec avis, ce
qui serait probablement un avis d'enquête dans le cas présent. De plus, comme la servitude était
rattachée à Whiteacre, Olivia pouvait continuer à l'utiliser. Une servitude par prescription, réponseA,
ne fonctionne pas car l'usage était permissif. Une servitude implicite par nécessité ou implicite par
usage antérieur ne fonctionnera pas non plus car il n'y avait pas de propriété commune des deux
parcelles. Ces théories reposent sur le fait qu'une servitude est implicite en tant que "partie de
l'accord" lorsque la propriété est divisée par un propriétaire commun. Cela ne s'est pas produit ici.
Les réponsesC etD sont donc incorrectes. Une servitude légale par nécessité pourrait être une
possibilité, mais ce n'est pas un choix répertorié.
Lecture recommandée :
Pages 492-496 de Dukeminier, Property, Concise Edition 2E
6

Olivia était propriétaire d'un terrain bordé au nord par Green Road et au sud par Brown
Road. En 1996, Olivia a construit une maison sur la moitié nord de la propriété, qu'elle a
appelée Greenacre. Pour se rendre à la maison, elle empruntait parfois l'allée principale
menant au nord à Green Road, mais après de fortes chutes de neige ou de pluie,
Green Road n'était accessible qu'avec un véhicule à quatre roues motrices. C'est
pourquoi elle utilisait souvent la "voie détournée", un chemin de terre traversant la
moitié sud de la propriété, qu'elle appelait Brownacre, et menant à Brown Road. En
2006, Olivia a vendu Greenacre à Andrew et elle a construit et emménagé dans une
maison à Brownacre. Bien que l'acte ne mentionne pas de servitude sur Brownacre,
Andrew utilisait fréquemment le chemin de terre pour rejoindre Brown Road, qui était un
moyen plus rapide de se rendre sur son lieu de travail. Olivia le voyait souvent et le
saluait. En 2014, une coulée de boue a fermé Green Road, et les autorités du comté
ont informé Andrew que la route était gravement endommagée et qu'elle risquait de ne
pas être réparée avant des années. Par la suite, Andrew a utilisé exclusivement le
chemin de terre au-dessus de Brownacre pour entrer et sortir. En 2016, Olivia a vendu
Brownacre à Betsy. Peu après, Betsy a installé un portail verrouillé en travers du
chemin de terre et a interdit à Andrew de l'utiliser. Dans cette juridiction, le délai de
prescription est de 10 ans. Laquelle des hypothèses suivantes est la meilleure théorie
d'Andrew pour une servitude sur Brownacre ?

(A) Préclusion

(B) Implicite par nécessité

(C) Prescription

(D) Implicite par utilisation antérieure

Raison d'être :
La bonne réponse estD. Une servitude implicite par usage antérieur nécessite un propriétaire
commun (Olivia), un usage antérieur (elle utilisait fréquemment le chemin de terre menant au
chemin Brown), une nécessité raisonnable (plus facile à satisfaire qu'une nécessité stricte et
probablement satisfaite ici en raison des difficultés rencontrées en hiver) et un avis (le chemin de
terre serait visible). Un acheteur raisonnable serait-il fondé à supposer que l'utilisation de la piste en
terre battue était incluse dans le contrat ? Ce n'est peut-être pas gagné, mais c'est la meilleure
solution pour Andrew. L'autre possibilité est une option qui n'a pas été mentionnée : une servitude
légale par nécessité, qui obligerait Andrew à indemniser Betsy pour l'utilisation de la voie ferrée et
dépendrait des exigences légales de la juridiction dans laquelle le terrain est situé. La réponseA est
incorrecte car une servitude par préclusion nécessite une confiance raisonnable dans une promesse
faite par Olivia (ou Betsy) ; en l'occurrence, il n'y a pas eu de telle promesse. On pourrait penser
qu'une servitude implicite par nécessité serait possible, réponseB, étant donné qu'Andrew n'a aucun
moyen réel de quitter sa propriété, sauf par Brownacre. Toutefois, une servitude implicite de
nécessité exige que la nécessité soit apparue au moment où la propriété a été divisée en deux par
le propriétaire commun. En 2010, lorsqu'Olivia a vendu Greenacre à Andrew, ce dernier a pu utiliser
Green Road. Les faits indiquent que c'était parfois difficile - il fallait un véhicule à quatre roues
motrices - mais les tribunaux exigent généralement une stricte nécessité, ce que ces faits ne
confirment pas. La prescription, réponseC, n'est pas bonne non plus ; l'utilisation indésirable n'a
probablement pas commencé avant 2016. Avant cela, l'utilisation était probablement permissive car
Olivia voyait Andrew utiliser la piste et ne semblait pas y voir d'inconvénient.
Lecture recommandée :
Pages 497-504 de Dukeminier, Property, Concise Edition 2E
7

Olivia était propriétaire d'un terrain bordé au nord par Green Road et au sud par Brown
Road. Il y avait une maison sur la moitié nord de la propriété, qu'Olivia appelait
Greenacre, avec une allée menant à Green Road, et un chemin de terre, sur la moitié
sud de la propriété, appelé Brownacre, menant à Brown Road. En 2006, Olivia a vendu
Greenacre à Andrew. Bien que l'acte ne mentionne pas de servitude sur Brownacre,
Andrew utilise fréquemment le chemin de terre. Après qu'une coulée de boue a fermé la
Green Road en 2014, qui est restée fermée pendant des années, Andrew a utilisé
exclusivement le chemin de terre au-dessus de Brownacre. Olivia a ensuite vendu
Brownacre à Betsy en 2016, qui a installé une clôture en travers du chemin de terre
pour empêcher Andrew de l'utiliser. Dans cette juridiction, le délai de prescription est de
10 ans. Andrew a intenté un procès pour pouvoir continuer à utiliser le chemin de terre,
et un tribunal a déterminé qu'Andrew avait une servitude légale sur Brownacre. Le
comté a finalement réparé et rouvert Green Road, et Andrew a cessé d'utiliser le
chemin de terre. Quelques mois plus tard, Betsy lui a demandé s'il avait l'intention de
réutiliser la piste, ce à quoi Andrew a répondu "Non". Deux ans plus tard, une autre
coulée de boue a fermé Green Road, et Andrew a voulu utiliser à nouveau la servitude.
Parmi les éléments suivants, quel est le meilleur argument de Betsy pour démontrer
que la servitude a pris fin ?

(A) Abandon

(B) Préclusion

(C) Libération

(D) Prescription
Raison d'être :
La bonne réponse estA. L'abandon nécessite une période de non-utilisation accompagnée d'une
intention d'abandon. Ici, Andrew a cessé de l'utiliser pendant une période considérable et a indiqué
son intention de l'abandonner en disant qu'il y renonçait. Il ne s'agissait pas d'une décharge, car une
décharge doit être écrite et signée par Andrew pour satisfaire au Statute of Frauds, ce qui rend la
réponseC incorrecte. La préclusion pourrait mettre fin à la servitude si Betsy avait changé de
position en se fiant à la promesse d'Andrew de cesser de l'utiliser, mais rien n'indique qu'elle ait fait
quoi que ce soit à son détriment. La réponseB est donc incorrecte. Enfin, une servitude peut être
résiliée par prescription, mais cela exigerait que Betsy bloque l'utilisation de la servitude par Andrew
pendant la période de prescription qui, nous le savons, est de dix ans. Cela ne s'est pas produit ici.
La réponseD est donc incorrecte.
Lecture recommandée :
Pages 523-533 de Dukeminier, Property, Concise Edition 2E
8

Urban Development, Inc. a demandé à la ville de Maryville de modifier le zonage d'un


terrain pour le faire passer de R-1 (résidentiel unifamilial) à R-3 (résidentiel
multifamilial), afin de pouvoir faire disparaître des habitations plutôt délabrées et de
construire de nouveaux appartements et condominiums. La ville craignait toutefois de
déplacer un grand nombre de résidents à faibles revenus. La ville a donc autorisé le
changement de zonage en échange d'une clause restrictive exigeant qu'au moins 25 %
des unités de logement disponibles dans le nouveau projet soient réservées à des
résidents à faibles revenus. Urban Development a eu des difficultés à financer le projet
et, deux ans plus tard, a vendu l'ensemble du site à High End Housing, Inc. qui n'a pas
l'intention de respecter la convention. Lequel des points suivants est lemoins
susceptible de constituer un problème pour l'application du pacte juridique à l'encontre
de High End ?

(A) La charge de la convention ne peut pas être répercutée sur les propriétaires
ultérieurs de l'aménagement, car l'avantage est brut.
(B) La charge de la convention ne peut pas s'étendre aux propriétaires ultérieurs de
l'aménagement, parce qu'elle ne peut pas "toucher et concerner" la terre.

(C) La charge de la convention ne peut pas être répercutée sur les propriétaires
ultérieurs du lotissement, parce qu'il n'y a pas de lien de connexité horizontale entre la
ville et Urban Development, Inc.

(D) La convention peut ne pas être applicable parce qu'il n'y a pas de lien de
connexité entre la ville et High End.

Raison d'être :
La bonne réponse estD. La connexité horizontale est requise (entre Urban Development et la ville),
et la connexité verticale est requise (entre Urban Development et High End), mais la connexité n'est
pas requise entre la ville et High End. La réponseA est incorrecte car certaines juridictions
continuent d'appliquer l'ancienne règle anglaise qui interdit de faire courir la charge d'une convention
lorsque le bénéfice est détenu en brut. Ces tribunaux, et certaines lois, exigent qu'il y ait un
avantage direct pour un autre bien, ce qui n'est pas le cas ici. Les commentateurs dénoncent
généralement cette règle comme anachronique, mais on dit qu'elle persiste. Voir Thomas E.
Roberts, "Promises Respecting Land Use-Can Benefits Be Held in Gross", 51 Mo. L. Rev. (1986). La
réponseB soulève la question de savoir ce que nous entendons par l'exigence du "contact et de la
préoccupation". On peut dire que ce pacte touche et concerne la terre, car il est certainement lié à la
manière dont la terre est utilisée. Toutefois, on pourrait également faire valoir que cette mesure ne
modifiera pas réellement ce qui est construit sur le terrain, ni l'aspect des appartements, ni le
nombre de personnes qui y vivent, mais qu'elle ne fera que modifier les pratiques commerciales des
propriétaires en exigeant d'eux qu'ils qualifient 25 % des résidents en fonction de leurs revenus. Les
tribunaux pourraient justifier l'une ou l'autre conclusion, ce qui en fait au moins un argument possible
en faveur du haut de gamme. La réponseC est également un argument possible pour High End car,
en vertu des règles traditionnelles, il n'existe pas de lien de connexité horizontale entre la ville et
Urban Development. Si cette juridiction adhère toujours à l'exigence de connexité horizontale, le
tribunal devrait conclure qu'il n'y a pas de connexité entre le concédant et le concessionnaire, de
connexité mutuelle ou de connexité entre le propriétaire et le locataire en l'espèce.

Vous aimerez peut-être aussi